Вы находитесь на странице: 1из 98

Seminar 1: Introduction to tort and

introduction to the tort of negligence


Donoghue v Stevenson [1932] (HL)
Mrs Donoghue went to a cafe with a friend. The friend brought her a bottle of ginger beer and an ice
cream.. Mrs Donoghue poured half the contents of the bottle over her ice cream and also drank some
from the bottle. After eating part of the ice cream, she then poured the remaining contents of the bottle
over the ice cream and a decomposed snail emerged from the bottle. Mrs Donoghue suffered personal
injury as a result. She commenced a claim against the manufacturer of the ginger beer.
Held: Her claim was successful. This case established the modern law of negligence and established the
neighbour test.
Anns v Merton LBC [1978] (HL)
Facts: The claimants were tenants of a block of flats built in accordance with the pla ns approved by the
council. The foundations were too shallow. The tenants sued for the cost of making the flats safe on the
basis that the council either negligently approved inadequate plans or failed to inspect the foundations
during construction.
Judgment: A DoC was owed by the council and that if their inspectors did not exercise proper care and
skill then the council was liable even though the loss suffered was economic loss.
Notes: two-stage approach to DoC formulated.

Seminar 2: Psychiatric Illness


Bourhill v Young - the event which actually happened would have caused psychiatric illness to a person
of sufficient fortitude or customary phlegm
McLoughlin v OBrian [1983] (HL)
Facts: Cs husband and children were involved in a road accident. C heard about it 2 hours later. She went
to the hospital where she learnt her younger daughter was dead and saw her husband and two other kids
severely injured, all still covered in oil and mud. She sued in nervous shock.
Judgment: The nervous shock suffered was the reasonably foreseeable result of the injuries to her family
caused by Ds negligence and she was entitled to recover damages.
Notes: For somebody not directly involved in the incident to recover in nervous shock, there has to be a
close relationship between C and the accident victims/ Further, C was to be in close proximity to the

accident in time and space. (Either present or witness the immediate aftermath. Here, Lord Wilberforce
seems to stress the importance of them not being cleaned up yet.)
PRIMARY AND SECONDARY VICTIMS
Alcock v Chief Constable of South Yorkshire [1992] (HL)
Facts: Concerns the Hillsborough disaster in 1989 where 96 Liverpool fans died in a massive crush. The
accident was caused by the police negligently allowing too many supporters to crowd in one part of the
stadium. A large number of claims were made by those present at the scene and those who had viewed the
events on TV. Claims were also made by people with varying relationships to those crushed.
Judgment: HL introduced the distinction of primary and secondary victims
Primary victims
-

here it was said they are those involved either mediately or immediately but that definition
changes
Primary victims have to establish that it was reasonably foreseeable (to D) that they would suffer
harm (it is unclear whether this covers psychiatric harm)

Secondary victims
Essentially, everyone not directly involved in the incident. Note that this criterion presupposes that
someone else has suffered physical harm as a result of Ds action and these victims have suffered some
psychiatric injury as a result of viewing the psychical harm. To sue, they need:
1.
2.
3.
4.

Proximity of relationship close tie of love and affection (some assumed; burden on C otherwise)
Proximity in time and space to the accident
Perception by own senses (sight or sound typically), not TP reports
There has to be shock (per Lord Ackner the sudden appreciation by sight or sound of a
horrifying event which violently agitates the mind)

CLASS OF PRIMARY VICTIMS:


White v Chief Constable of the South Yorkshire Police [1999] 1 All ER 1
Facts: In relation to the Hillsborough disaster but involves police suing their employer in negligence for
the PTSD they suffered as a result of the events. Note that the police in question were not those who were
at any risk of physical harm, nor did they suffer any. They were at the accident though.
Judgment: In Alcock, rescuers were also placed in the class of primary victims. However, here the HL
changed the definition and said to be a primary victim, you need to be in the zone of foreseeable danger.
(You must establish objective exposure to danger or a reasonable belief that there was an exposure to
danger)
Notes: In the judgment there was clearly concern to avoid different treatment of police officers and
bereaved relatives. Hence the exclusion of rescuers (though it was more of a middle ground than a total
exclusion). After white it seems that the HL had decided to create a very definite narrow category for
primary victims. But see W v Essex and Re Organ

Unwitting instrument of anothers negligence?(There is a further category of primary victims in situations


where C believes he has caused anothers death or injury. This would only succeed if C was actually
present when the death or injury occurred Hunter v British Coal [1998] 2 All ER 97 - proximity
requirement)
W v Essex County Council [2001] 2 AC 592(regarding a known child abuser being placed with foster
parents who specifically said they wouldn't accept such a child.) Ultimately, whats relevant here is that
HL said the category of primary victims not closed
Re Organ Retention Group Litigation [2005] QB 506- parents of deceased children whose organs were
retained after post-mortem examination treated as primary victims.
Notes: It is clear the parents were at no time in the zone of foreseeable physical danger. 2 reasons were
given for holding they were primary victims
1. Relationship between parent and health authority is almost analogous to that of doctor and patient
(Where it is well-established that a DoC exists)
2. In this situation, there isnt anybody who could count as a primary victim except the parents
therefore, the primary victims must be the parents. (Slightly dubious as it hinges on the fact that
the babies were not born as human beings)

PRIMARY VICTIM NEED NOT (ALWAYS) PROVE FORESEEABILITY OF PSYCHIATRIC INJURY


Page v Smith [1996] 1 AC 155
Facts: C was involved in a minor car accident and was physically unhurt in the collision, but the accident
caused him to suffer a recurrence of ME from which he had suffered for about 20 years but was then in
remission.
Judgment: HL held that foreseeability of physical injury (even if none occurs) was sufficient to allow a
primary victim to recover in psychiatric injury.
Grieves v FT Everard & Sons Ltd [2008] 1 AC 281 (Rothwell v Chemica & Insulating Co Ltd: Re
Pleural Plaques)
Facts: One of the claimants suffered a recognised psychiatric illness from the fear that he would contract a
serious asbestos-related illness in the future.
Judgment: HL refused to extend the principle from Page v Smith. Consider that C was working with
asbestos, he seems to be in the zone of foreseeable danger. Gave 2 reasons.
1. The risk of physical harm wasnt immediate (asbestos related diseases take years to manifest)
2. There was an intervening event between the asbestos exposure and the onset of the psychiatric
illness (namely, the medical report; he only knew hed gotten pleural plaques as a doctor had
examined him and the test results indicated their presence)
Notes: Mitchell thinks these reasons arent convincing and that the HL isnt completely sold by PvS and
thus are going to great lengths to distinguish it.
PRIMARY VICTIM S LIABILITY

Greatorex v Greatorex [2000] 1 WLR 1970


Facts: D was seriously injured when he negligently got into an accident. C, a fire officer and Ds father,
was called to the scene (in the course of his employment) and subsequently suffered PTSD as a result of
seeing his sons injuries
Judgment: A victim of self-inflicted injuries owes no DoC to a secondary victim who suffered a
psychiatric injury from witnessing the incident or its immediate aftermath. (i.e. no liability for primary
victim where he is the negligent defendant)
Employees
White v Chief Constable of the South Yorkshire Police [1999] 1 All ER 1 (as above) - no special
protection for employees as secondary victims
EMPLOYEES (AND PRISONERS )
CHECK
SUMMARY

Law generally looked upon personal injury claims more favourably than psychiatric claims
Idea being that were expected to deal with a certain amount of distress in life
This imbalance was somewhat addressed in McLoughlin where a woman who wasnt directly
involved as such was allowed to claim for psychiatric injury. Some requirements are imposed to
keep claims down though. Namely proximity of relationship and proximity of time and space.

This is taken further in Alcock where theres a distinction drawn, with regards to psychiatric
injury suffered, between primary and secondary victims.
Primary Victims (Per Alcock, those involved mediately or immediately and includes rescuers.
Changed in White to those in the zone of foreseeable danger)
Secondary victims claims subject to stricter conditions
o Proximity of relationship (burden on C. Some relationships would have this assumed.
E.g. parent-child, spouses)
o Proximity of time and space (witness event or immediate aftermath e.g. mcloughlin)
o Experience the event unaided; i.e. with your own senses (not on TV or through a TP.
E.g. McLoughlin she saw them before they were cleaned up. In Alcock, a claim by
someone who saw the primary victim in a morgue a few hours later, cleaned up as well,
failed)
o Shock (cant be a gradual onset)

Definition of primary victim refined in White. Becomes zone of foreseeable danger (note the
concern to avoid differential treatment between police and bereaved relatives.
The definition of primary victims may not be settled though
o W v Essex class of primary victims isnt closed
o Re Organ Parents treated as primary victims
Analogy with doctor-patient relationship
The need to have some primary victim

There is a further category of primary victim. This may be described as being an unwitting
instrument of anothers negligence. It applies in situations where C believes he has caused
anothers death or injury. This can only succeed if C was actually present when the death or
injury occurred. E.g. in Hunter, the owner of the mine was negligent in not having the minimum
safety clearance distance in the mine. C a driver struck a hydrant and when he was away getting
helped, someone else was killed by an explosion from the hydrant causing C to have a psychiatric
injury.

Also, it seems that with regards to psychiatric injury for primary victims, only some physical
harm needs to be foreseeable. (see Page v Smith)
But note how this was not followed in Grieves (RothwellI)
o Risk of physical harm wasnt immediate
o Intervening event (medical report)

Also, where the primary victim is the negligent defendant, he is not liable for any psychiatric
injury suffered by secondary victims (Greatorex v Greatorex)

Employees?

Seminar 3: Pure Economic Loss


(1) The concept of pure economic loss, and the general rule against recovery in
negligence
(A) THE DISTINCTION BETWEEN PURE ECONOMIC LOSS AND CONSEQUENTIAL ECONOMIC LOSS
Spartan Steel & Alloys Ltd v Martin & Co (Contractors) Ltd [1973] QB 27
Facts: Ds employees were digging up a road when they negligently damaged the electrical supply cable
to Cs factory, leaving the claimants without power for 14.5 hours. They scrapped a melt in the furnace,
reducing its value by 368 and they lost a profit from the sale of the metal from that melt of 400. They
could also have completed four further melts during the power cut and their loss of profit from those
melts was 1,767.
Judgment: They could recover the damage to the melt in progress (physical damage) and the loss of profit
from that melt (consequential economic loss). However, they could not recover for the loss of profit
during the time the electricity was off, as it was pure economic loss and not recoverable.
Notes: Broadly, we see 4 reasons against it
-

What people should tolerate


Worry about the size/extent of the claims and the ease of exaggeration
Sharing the burden (links to (2). Itd be very heavy for a single contractor to bear it)

There is a statutory immunity for supplies of utilities so when the disruption is caused by
someone else, it doesnt seem fair.

Also, a general idea against PEL being recoverable is that in the commercial context, people often seek to
inflict economic loss deliberately, let alone negligently.
Overseas Tankship (UK) Ltd v Morts Dock & Engineering Co (The Wagon Mound) (No 1) [1961] AC
388 this case is considered in more detail later in the course, in the seminar on remoteness of damage;
here the important thing is to understand the principle: where a physical injury leads to consequential
economic loss (such as loss of earnings), that consequential loss is recoverable provided that it was
reasonably foreseeable that loss of that type would occur.
(B) PURE ECONOMIC LOSS IN RELATION TO DEFECTIVE PRODUCTS (COVERED IN L&O 387-407
BUT WE WILL LECTURE YOU ON THIS MATERIAL )
Donoghue v Stevenson [1932] AC 562 In Donoghue, we see this idea emerging that tort duties may be
owed to ultimate consumers of a product even though these consumers do not contract with the
manufacturer.
Dutton v Bognor Regis Urban District Council [1972] 1 QB 373 (361,362)
Anns v Merton LBC [1978] (HL)In Anns, the HL held that a local authority may owe a DoC in
negligence in exercise of its powers of inspection under the Public Health Act 1936. Lord Wilberforce
clearly thought the loss was not purely economic. As a matter of classification, he was mistaken. The
damage suffered was PEL. No separate damage had been done to the property of the plaintiffs, other
than the building itself, by the defendnats alleged breach of duty. (We see the HL recognise this error and
thus depart from Anns in D&F)
D & F Estates Ltd v. Church Commissioners for England [1989] AC 177
Facts: The Church Commissioners owned a block of flats built by a firm of contractors. The plastering
work was sub contracted. Fifteen years after the property was built it was found that the plastering work
was defective. As there was no direct contractual relationship between the plaintiff and the defendants an
action was brought in tort.
Judgment: The HL held that tort duties under DvS are only owed in respect of damage done by the item
that is manufactured by the defendant. Lord Bridge attempted to reconcile this decision with Anns by
means of the complex structure theory.
Murphy v Brentwood District Council [1991] 1 AC 398
Facts: A council approved plans for a concrete raft upon which properties were built. The raft moved and
caused cracks in the walls of a property which was sold for 35, 000 less than it would have done if it
were not defective.
Judgment: The HL overruled Anns and held that the council was not liable in the absence of physical
injury.
Notes: The complex structure theory was technically dismissed but there were some qualifications to
this (i.e. situations where it might apply) mean the potential exists for complex arguments about the
application of Murphy:

Lord Bridge distinguished between the foundations (an integral part of the larger structure) and a
distinct item (e.g. a heating boiler)
Lord Keith argued that the components could not be treated separately if erected and equipped
by the same contractor On the other hand where, for example, the electrical wiring had been
installed by a subcontractor and due to a defect caused by lack of care a fire occurred which
destroyed the building there might be liability in tort.
Lord Jauncey also felt that integral components could only be treated as separate property if
installed by a different contractor. However, he thought these integral compenets could be
distinguished from the examples of the central heating boiler or electrical installations, which he
described as ancillary equipment and for which he thought normal principles could apply.

Further, it is clear in Murphy that even in the case of defects that cause actual damage to a separate
structure or indeed to the person, there will be liability only if the damage is caused by a defect that
remains latent. Once the defect is patent (becomes known), it no longer poses a danger.
This strict distinction was doubted in Targett v Torfaen Borough Council [1992] 3 All ER 27, and it was
suggested that the availability of reasonable steps to avoid the danger should be considered as well.
Defective Premises Act 1972 ss1, 2 Provides a statutory remedy against a person taking on work for or
in connection with the provision of a dwelling. The protection extends to subsequent purchasers and
cannot be excluded or limited via contract. However, it only applies to dwellings and an action must be
brought within 6 years of completion of the work. (and local authorities are not covered by the a person
definition)
Winnipeg Condominium Corporation No 36 v Bird Construction Co [1995] 1 SCR 85
Bryan v Maloney (1995) 69 ALJR 375
-

It does seem that Murphy was decided more on policy reasons


o Anns created a new form of liability best left to parliament
o Argument that consumer is already protected by other means. Specifically, first party
insurance
o There is already consumer protection by statute
And the above 2 cases illustrate how other jurisdictions have seen fit to depart from Murphy

(2) The special principles governing duty of care


(A) ASSUMPTION OF RESPONSIBILITY
Hedley Byrne & Co Ltd v Heller & Partners Ltd [1964] AC 465
Facts: The appellants were advertising agents, who had placed substantial forward advertising orders for a
company on terms by which they, the appellants, were personally liable for the cost of the orders. They
asked their bankers to inquire into the company's financial stability and their bankers made inquiries of
the respondents, who were the company's bankers. The respondents gave favourable references but
stipulated that these were "without responsibility." In reliance on these references the appellants placed
orders which resulted in a loss of 17,000. They brought an action against the respondents for damages
for negligence:Judgment: a negligent, though honest, misrepresentation, spoken or written, may give rise to an action for
damages for financial loss caused thereby, apart from any contract or fiduciary relationship, since the law

will imply a duty of care when a party seeking information from a party possessed of a special skill trusts
him to exercise due care, and that party knew or ought to have known that reliance was being placed on
his skill and judgment. However, since here there was an express disclaimer of responsibility, no such
duty was, in any event, implied.
Notes: This case gave rise to the Hedley Byrne test. Namely, a DoC may arise in respect of a loss where:
1. (Voluntary?) assumption of responsibility by D (where D exercises skill and judgment knowing C
will rely on the statement)
2. Reasonably reliance by C
(I) THE CAPARO TEST
Smith v Eric S Bush (a firm) [1990] 1 AC 831
Facts: Here, D gives a disclaimer with their advice which if anything was clearer than the HB disclaimer.
HL said that it failed the requirements of UCTA. (1977) and HB was decided in 63. So at the time of hb
HL didnt have to worry about UCTA.
Judgment: even though the surveyor said he wasnt assuming responsibility, that disclaimer wasnt
reasonable in the framework of UCTA and it was to be disregarded and what you were left with is a
person who had assumed responsibility even though he said he hadnt. (so note the lack of requirement of
a voluntariness requirement)
Notes: Lord Griffiths in his judgment set out an alternative formulation by which to judge whether a duty
of care should be recognised as arising in such a case (which went back to the general conception and
essentially is the framework of the Caparo test), avoiding the terminology of assumption of
responsibility. It should be noted that the search for a voluntary assumption of responsibility was
intended by Lord Devlin in Hedley Byrne to be a way to judge whether proximity was present. However,
the reasons Lord Griffiths ultimately give seem to fit with the old test. This was perhaps a case in which
an alternative approach to justifying the DoC was simply more appropriate. Nonetheless, Lord Griffiths
attempt to make the assumption of responsibility fit the facts amounted to a distortion of the concept and
has caused significant confusion.
Caparo Industries Plc v Dickman [1990] 2 AC 605
Facts: Caparo owned shares in a TP company. D audited the TP company and reported a high profit when
there was actually a loss. On the basis of the report, Caparo decided to increase its investment and launch
a subsequent takeover bid. When the mistake was discovered, Caparo tried to claim against D for the loss
they incurred by paying a higher price for the new shares they purchased as a result of the negligently
produced report.
Judgment: It was held that the defendant audit company did not owe a duty of care in this respect. They
owed it to the shareholders at large as a body to ensure that the company was run properly. If any duty
was owed to individual shareholders, it was only in respect of losses they might make in relation to their
existing stake in the company namely by selling undervalued shares and most certainly not to losses
resulting from the purchase of additional shares which would result from a wholly independent
transaction having no connection with the existing shareholding

Notes:Caparo sets out three criteria for the existence of a Duty of Care
1. Foreseeability
2. Proximity of relationship
3. Fair, Just, and Reasonable to impose liability
After Caparo, assumption of responsibility appeared to have been fatally weakened as a justification for
the DoC in cases of PEL. Certainly it was clear that the duty was not to be understood as voluntarily
assumed but note the resurgence of the term in the later cases (below).
(II) THE INTER-RELATION BETWEEN THE TESTS
Merrett v Babb [2001] 3 WLR 1 The CA doubted whether assumption of responsibility has any
meaning. Here they said the test has merged with Caparo. Indeed, in 2 other CA cases prior to this saw
them treating the assumption of responsibility and Caparo as alternative tests, applying both arguing
they should lead to the same result.
Customs and Excise Commissioners v Barclays Bank plc [2007] 1 AC 181 (disapproving the approach
taken by the CA, which is critically analysed in P Mitchell and C Mitchell (2005) 121 LQR 194)
Facts: C had obtained freezing orders and served them on D. The purpose of these orders was to prevent
two companies from removing funds from their bank accounts, so that C could recover outstanding VAT
from those accounts. D failed to take action to prevent funds from being moved out of their accounts. It
was alleged that this failure was negligent. C could thus not recover the full sum they were owed and
sought to recover the shortfall from D on the basis that it breached a DoC owned to them to abide by the
orders.
Judgment: The HL gave 5 slightly different judgments (same conclusion) Steele explains the judgment as
follows
1. The first stage in deciding a novel case of economic loss is to ask whether there is a voluntary
assumption of responsibility
2. In an assumption of responsibility is established, this may be sufficient (in other words, there may
be no need to consider policy issues)
3. At least two of the judges seem to have treated assumption of responsibility as an aspect of
proximity (Lord Hoffman is particularly clear on this)
o If this is correct, then assumption of responsibility forms part of the three stage test,
rather than a narrower test to be applied first. Hoffman hints that where there is an
assumption of responsibility, it is the nature of the relationship which makes the duty
FJR.
4. No assumption of responsibility could be established here
o Even if the test of the existence of an assumption of responsibility is objective, a degree
of voluntariness is essential. Here, there was an adverse relationship between the parties.
5. This was not the end of the matter, Caparo was applied. So the HL seems to recognise that
Caparo is a wider test. Indeed, this helps to explain Smith v Bush as there, despite the lack of
assumption of responsibility, there was proximity and foreseeability (and policy)
6. Policy issues were decisive against a duty of care
7. Lord Hoffmans short-cut: no common law duty of care could be said to arise out of the freezing
order itself (draws an analogy with omissions and Stovin v Wise)
(III) EXPRESS ASSUMPTIONS OF RESPONSIBILITY

It seems that assumptions of responsibility can be deduced either from the general nature of the
relationship (Henderson), or if the general nature of the relationship is not compatible with such a
deduction it may be deduced from specific words, conduct or circumstances which (Williams) override
the general features of the relationship.
Williams v Natural Life Health Foods [1998] 1 WLR 830 - in determining whether there had in law
been such an assumption an objective test was to be applied (in reality this doesnt crop up often.
Nonetheless, it seems clear that the assumption of responsibility no longer need be voluntary) , the
primary focus being on things done or said by the defendant or on his behalf and the question being
whether the plaintiff could reasonably have relied and had relied on an assumption of personal
responsibility by him; and that the fact that the brochure given to the plaintiffs had held the company out
as having the expertise to provide reliable advice to prospective franchisees and had made it clear that that
expertise derived from the second defendant's experience in the health food trade was insufficient to
render the second defendant personally liable to the plaintiffs
Calvert v William Hill Credit Ltd [2008] EWHC 454 (Ch) - Here, the court says that by confirming that
C would be prevented from telephone betting in response to C's request for self-exclusion, W had
assumed responsibility towards C as the relationship was akin to contract, save for consideration.
(Nonetheless they werent liable as it was held that D could not have been said to have caused Cs losses
as C would have in any event ruined himself)
(IV) ASSUMPTIONS OF RESPONSIBILITY IMPLIED FROM THE RELATIONSHIP
These cases also really highlight how Hedley Byrne liability has moved beyond negligent misstatements
(and indeed, beyond PEL)
Spring v Guardian Assurance Plc [1995] 2 AC 296
Facts: Concerns a negligently prepared reference (by D) resulting in C not being hired for another job.
Judgment: Two of the majority interpreted Caparo in reaching their decision (Lord Woolf and another).
Lord Goff however based his decision through a specific application of principles drawn from Hedley
Byrne. (with whom Lord Lowry agreed)
Notes: Importantly, from Lord Goffs reasoning, we see the following:
There are number of factors the courts take into consideration to decide theres been an implied
assumption of responsibility;
-

The nature of the relationship is absolutely key. There are certain relationships such as solicitorclient and doctor-patient that the courts regard as giving rise to the implied assumption of
responsibility.
This conclusion is due to the special skills these professionals possess.
In relation to these special skills, if D holds themselves out as having some special skill which
theyre going to exercise (for the benefit of C), it leads to an implicit assumption of responsibility.
It is not necessary for D to hold themselves out as merely having a special skill; it could be
knowledge as well (as with knowledge of C regarding the reference)

Henderson v Merrett Syndicates Ltd [1995] 2 AC 145

Facts: The case arose out of losses suffered by investors (referred to as Names) suffered by the Lloyds
insurance market in London during the 1980s. This involved a situation where the claimant wanted to join
an insurance syndicate. Hed hire an agent (either a members agent or a combined agent acting as a
members agent) to generally advise him and place him on the syndicate etc...
The members agent would be the one who contracted with the managing agent (or combined agent acting
as a managing agent) who managed the syndicate. As can be seen, there are a variety of contracts in play
here.
Judgment: Lord Goff (giving the leading judgment) holds there could be liability in tort and emphasised
the concept of assumption of responsibility drawn from Hedley Byrne. (He builds on his own judgment in
Spring). LBW agreed the central concept was assumption of responsibility but placed considerable
emphasis on fiduciary duties as forming the historical basis for the action in Hedley Byrne.
Notes: with regards to concurrent liability in contract and tort, he saw no issue, the practical result being
that a claimant could choose a remedy which appears to him to be most advantageous
White v Jones [1995] 2 AC 207
Facts: Here, a testator executed a new will after a family quarrel, disinheriting his two daughters, the
plaintiffs. After reconciliation, he contacted his solicitors with instructions to draw up a new will,
restoring the legacies to the plaintiffs. Little progress was made and the testator died before the new will
was completed.
Judgment: Lord Goff acknowledged there was no trust assumption of responsibility on the part of Ds
towards Cs. In his judgment, the case concerned a wrong which required a remedy and the best way of
providing it was to hold that the assumption of responsibility which existed between Ds and the testator
extended to Cs.
Notes: In terms its treatment of legal principles, White v Jones is slightly controversial. On the other hand,
the solution was intended to be confined to negligence in respect of wills. However, it has been also been
applied in the different but analogous situation of advice in respect of pension rights, where the defendant
advisor is aware that the client intends to make provision for his or her dependants (Gorham v British
Telecommunications plc [2000] 1 WLR 2129)
Williams v Natural Life Health Foods [1998] 1 WLR 830
SUMMARY:
-

The law doesnt like claims for PEL (Spartan Steel)


o We specifically try to cause economic loss often
o Idea of what we should bear
o Burden sharing theory
o There might be unlimited liability

- Broadly, there are 4 categories we see PEL


1. relational economic loss (economic loss caused by damage to property of another party e.g.
Spartan Steel
General rule against liability
2. Economic loss caused by acquiring a product that turns out to be defective (Murphy)

Generally, theres no liability but note the room for argument in murphy
3. Economic loss caused by reliance on negligent misstatement (Hedley Byrne)
4. Extended Hedley Byrne Liability (Cases after hb)

With regards to category 3, we see in HB this idea of:


o A voluntary assumption of responsibility (making a statement knowing it will be relied
on)
o Reasonable reliance on the statement

In Smith v Bush, this idea seems to be distorted and a new test is suggested to fit the facts
This essentially becomes the 3-stage Caparo test
o Foreseeability (harm)
o Proximity (relationship)
o FJR (liability)

In later cases however, we see a resurgence of this assumption of responsibility idea (see
especially Lord Bridge in Spring, Henderson and White). Nonetheless, it is now clear that the
assumption is not voluntary, but objective (see especially William)

There was thus a conflict then in the test for a DoC in relation to economic loss which had to be
resolved. The CA wrestled with the difficulty, saying that they were alternative tests which should
lead to the same result and even going so far as to say the assumption of responsibility had
merged with Caparo (Merrett)

Broadly, after Customs and Excise v Barclays, it seems you apply the assumption of
responsibility first and that may be sufficient (though there is debate over whether you are
applying one test first followed by the wider one or whether the tests have indeed merged but a
finding of this assumption means that it is FJR). Should that fail, you apply the wider Caparo test.

So after Customs excise, in general with regards to PEL:


1. There is actually no overarching exclusionary rule for PEL but rather, two specific exclusions for
categories 1 and 2. Even then, the exclusion for category 1 can seem arbitrary and while category
2 has more concrete reasons, note that they are doubted and also, the issues with Murphy
2. Outside the scope of the exclusionary rules, additional criteria will apply in addition to
foreseeability.
3. If a voluntary assumption of responsibility is present, it may suffice without separate
consideration of policy issues
4. If the assumption of responsibility is not present, the three stage Caparo test may nevertheless be
satisfied. But note that in such cases, policy considerations will be especially important. (per Lord
Bingham in Customs Excise the fair, just, and reasonable criterion can be frankly and openly
reassessed in terms of policy

Seminar 4: Duty of Care 3 Omissions


(1) Policy reasons for limiting liability for a failure to act
Stovin v Wise [1996] AC 923
Facts: C was on a motorcycle and involved in an accident with D. D claimed accident was caused partly
because her view was obstructed (local authority was a joined D). Local authority had written to the
'obstructors' (people owning the things obstructing the view) asking if they could take the necessary
action. They received no reply and proceeded to do nothing for a year. They were held 30% liable and
appealed to the HL.
Judgment: In general, no liability for omissions without a special duty to act. Lord Hoffman gives 3
specific reasons.
1. Political reason (restriction on individual autonomy; but, doesnt seem to weigh up competing
factors)
2. Moral reason (why pick on me argument; again, individual concern overrides others. Also,
sometimes there are reason to pick on someone e.g. as possessing a particular skill)
3. Economic efficiency (activities should bear their own costs; but sometimes it may require a small
act to save someones life. Is that less efficient? This argument only seems to apply in the
business context. E.g. letting a child drown wont help him be more efficient in the future)

(2) Distinction between acts and omissions

It can be difficult to distinguish the two. A reasonable working test is to look at the process of causation.
The key thing to look out for is whether the defendants contribution made things worse than they were or
whether the defendant is merely not making things better. Were concerned with the latter here.

(3) Examples of liability for omissions


(A) CREATING THE RISK
e.g. Hobbs (Farms) Ltd v Baxenden Chemical Co [1992] 1 Lloyd's Rep (in leni)
C is injured by a product and this product had been made by the D perfectly carefully. At the time no one
realised there was anything dangerous or problematic with it. Later however, problems with design were
brought to the manufacturers attention but the manufacturer made no attempt to contact purchasers. Duty
of care was owed in regard to that negligent failure to warn the purchasers of that product about that
defect that had come to light.
(B) UNDERTAKING RESPONSIBILITY FOR THE CLAIMANT S WELLBEING
Capital and Counties plc v Hampshire County Council [1997] QB 1004
Facts: 3 appeals held all essentially concerned with the fire brigades liability for failing to put out a fire
in some way. In the second 2 cases, all that we could say about the fire brigade was that they hadnt done
enough. Whereas in the first one, the fire brigade arrives, apparently they dont do enough to start with,
but then the fire brigade officer orders the sprinkler system turned off and that made the building burn
down.
Judgment: In general, theres no DoC owed by a fire brigade in respect of a fire. Nonetheless, in the third
case here there was liability.
Notes: There are no policy reasons to require a duty being imposed in respect of getting there in time once
they accept a call. Further, once they arrive, the fact that there may be conflicting demands placed on the
fire services (e.g. might have to break down another house to save a block) means there shouldnt be a
duty imposed there either.
Kent v Griffiths [2001] QB 36
Facts: An ambulance had not turned up in time for unexplained reasons. The result of their delay was
severe injury to the claimant.
Judgment: There is no general duty on the ambulance service to respond to a call; but once a call has been
accepted, the service owes a duty to the named individual at a specific address.
Notes: CA distinguish the fire service because conflicting demands arent placed on them once theyre
allocated to a particular call. Lord Woolf however recognises they if theyre called to the scene of a major
road accident for instance, there might still be conflicting demands (e.g. who to save first) but says that a
duty of care would still exist. (and tort law would avoid imposing excessive liability as long as they
werent negligent in choosing the order in which they treated people) This distinction between Capital
and Kent does not seem convincing.

Van Colle v Chief Constable of the Hertfordshire Police [2009] 1 AC 225, [55] (Lord Bingham)(Case
where C was threatened with death if he did not drop charges and eventually killed) Here, Bingham
suggests that the real distinction between the two cases is that Capital is about property damage while
Kent is about personal injury. Life should be given more weightage but beyond that, property, especially
commercial property as in Capital, is typically insured. Though courts arent meant to take that into
account, they often do. Flowing from this, it does seem to suggest that if a fire brigade is required to help
someone personally in danger, there could be a DoC.
Barrett v Ministry of Defence [1995] 1 WLR 1217
Facts: C got very drunk on cheap duty free booze at his naval bases bar and lies down. A senior officer
organises for him to be taken away and hes left alone and proceeds to puke and choke.
Judgment: It is held that once the duty officer organises for him to be taken away, a duty of care arises.
Notes: There was a question of whether a duty should arise for the Ministry of Defence with regards to
preventing him from becoming so drunk as to become a danger. The answer seems to be no as the law
favours autonomy and the effect of such a duty would be to dilute self-responsibility. (So the duty here
is relatively limited. Cf. Jebson)
Jebson v Ministry of Defence [2000] 1 WLR 2055
Facts: Soldiers go to the town of Portsmouth. Transport is a lorry with a canvas back basically, on the way
back, the soldiers notice that driving along behind the lorry is a young woman in the car. In an effort to
impress her, one of them climbs up to the lorry roof to do a dance. He falls off and gets injured.
Judgment: Because the officer organised the event, he assumed a responsibility for the safety of the
soldiers throughout. So the mere fact that he was the CO who organised the event meant that he was held
to have responsibility throughout.
Notes: Is it because of his position in employment rather than the mere fact he organised an event? Is it
even an employment context? Does Jebson mark a large, and indeed perhaps overly large, extension of
this assumption of responsibility context? Consider the case of someone organising an office party, could
he be liable? The ruling seems confined to the military context.
Reeves v Commissioner of Police for the Metropolis [2000] 1 AC 360 - Man commits suicide in a cell.
Authorities knew he was at risk and failed to take steps to prevent him from killing himself. There itd
seem that the position of responsibility occupied by prison authorities is what gives rise to the duty. It is
because they locked someone up, confined his movements etc..., that they ought to take careto provide
safety in that confined space.

(C) O CCUPATION OF LAND


Its said that a persons control over land gives rise toa responsibility to prevent damage occurring to other
people.

Goldman v Hargrave [1967] 1 AC 645


Facts: A tree on a mans land got hit by lightning. He thought hed let it burn out (thats the negligent bit.
Not putting it out). But the weather got worse and the wind changed etc so it flared up and the fire spread
to the neighbour.
Judgment: Hes made liable for failing to prevent the damage occurring to his neighbour.
Notes: Why should occupying land give rise to such a duty?
-

The person at risk isnt allowed to come onto your land to get rid of your danger also the person
cant really move away.
Note that there is an issue of cost of prevention. Its quite important on the facts there that theres
quite a simple task he could have done to stop the spread of the fire. So it seems were only
requiring landowners to take reasonable steps to prevent the harm rather than all possible steps.
However, as different people have different amounts of resources, there is a lingering question of
whether these reasonable steps will be assessed subjectively or objectively. Particularly with
omission situations where potentially expensive acts are required.

(4) Liability for Acts of Third Parties


(A) G ENERAL RULE
The general rule is that you dont owe a doc to prevent TPs from causing damage to anyone. Indeed, its
reinforced by the general rule of causation which is that a TPs voluntary act breaks the chain of
causation. So even if I do something negligent, e.g. leave the keys in my car so a thief steals it; if the thief
runs over you, his voluntary act would break the chain of causation b/w me leaving the keys and your
damage. So generally the fact that a TP has injured you cannot make me liable in negligence. (Essentially
describes Topp v London Country Bus [1993] 1 WLR 976)
But there are exceptions and the way theyve developed underlines an underlying difficulty in the law
regarding the DoC and thats whether you can use the foreseeability test or whether you have to identify
particular situations where a DoC arises. Nonetheless, as with other areas of tort, it is now reasonably
clear that mere foreseeability isnt enough.
(B) EXCEPTIONS TO THE GENERAL RULE
Home Office v Dorset Yacht [1970] AC 1004
Facts: Young offenders at Dorset snuck out, stole a boat and crashed into one owned by D.
Judgment:
Lord Reid: DoC should be based on foreseeability and if it was foreseeable, liability would hinge on
causation (and whether it was broken)
Lord Diplock (and Pierce): Mere foreseeability isnt enough. Identifies certain features of the situation at
hand that made it appropriate to impose a DoC. Here, the high degree of control justified the duty (so this
applies more generally to prison officers and prisoners)

Notes: So it seems an important factor in imposing liability for acts of Third Parties is the degree of
control D was entitled to exercise over the TP.
Smith v Littlewoods Organisation Ltd [1987] AC 241
Facts: Case concerned a cinema in the town centre of a Scottish town which was burned down by vandals
who broke in. Owners of the cinema didnt realise there was such a risk; when it burns down, it damages
neighbouring property and it is the neighbours suing the cinema owners.
Judgment: No DoC owed. Majority focus on the question of foreseeability.
Lord Goff by contrast was clear that mere foreseeability was not enough. Some categories he suggested
included:
-

Degree of control exercised over the TP


Presence of an assumption of responsibility by D to protect C against a TP
If D negligently created a risk which a TP then foreseeably triggered (e.g. fireworks for a village
festival stored in an unlocked shed which are then set off by trespassers causing damage)

Mitchell v Glasgow City Council [2009] 1 AC 874


Facts: C (Mitchells spouse) has this violent antisocial neighbour who causes him various problems.
Mitchell complains to the council. Council summoned neighbour to a meeting. Following the meeting
neighbour returns to the house and kills Mitchell.Here it was said that the council had been negligent as
they should have warned C that the meeting was going to take place. What would he have done then?
Gone out or maybe gone away for a few days to let the neighbour calm down for a bit.
Judgment: HL held that no DoC was owed to him. The idea being that they were simply exercising their
statutory powers andno criticism could have been made for them calling the meeting. They also said that
the council had not assumed responsibility to Mitchell to protect him against the neighbour and Lord
Hope says its only when theres been an assumption of responsibility that you can find that a failure to
warn a TP has been a breach of a DoC.
Notes: This case illustrates how important it is that we choose one option over another in terms of
imposing a DoC. Here, mere foreseeability would have led to the imposition of a duty. Focus on Goff in
Smith and the HL here to get an understanding of liability for acts of TPs.
Part of Lord Hopes reasoning was that if you imposed a duty here, youd have to impose a duty on
private landlords and housing associations
SUMMARY
-

No general duty to act in English law (Stovin v Wise)


A good way to tell the difference between an act and omission is whether the defendant made
things worse (which would be an act and thus potentially have liability) or whether he merely did
not make things better

There are nonetheless exceptions to this rule such as when you undertake responsibility to act
o This is explicitly not the case for the fire department (Capital and Counties)
No policy reasons to impose duty at time of answering call
Policy reasons (conflicting interests) against duty at time of arrival
Unstated reason? insurance

o
o
o
o
o
o
o
-

However, it seems to be so for the ambulance service (Kent)


No conflicting interests once allocated to a call. Nonetheless, even if there are
thered still be a duty
The distinction between the above 2 is unsatisfactory and in Van Colle, Bingham suggests
that the reason may be that one deals with personal injury and another with property. So
is there a duty if the fire service is called to rescue someone?
Where someone is clearly in a very bad state and you make some motion of helping them
(Barrett)
Perhaps employers and employees? (wide reading of Jebson)
Something in the nature of organising an activity for servicemen (narrow reading of
Jebson)
Prison officers and prison guards (Reeves)
Occupiers of land and visitors (Goldman. Note how special cost considerations may arise
here)

With regards to liability for the acts of third parties again, the general rule is that there is no DoC
owed. However there are some exceptions.
Home Office v Dorset Yacht; where D has a high degree of control over the TP. Note that the
category approach was favoured by the majority here.
Smith v Littlewoods; The majority favoured a foreseeability approach. Lord Goff firmly took a
category approach and suggested a few.
o High degree of control
o Assumption of responsibility
o Where D has created a dangerous situation but is negligent allowing the TPs to realise
the danger (or make the danger materialise essentially)
Mitchell; adopted Lord Goffs reasoning in Smith. So it seems the mere foreseeability approach is
gone.

Seminar 5: Duty of care 4: public


authorities
(1) The techniques used to narrow the scope of duty
(A) DIRECT LIABILITY AND VICARIOUS LIABILITY
Phelps v Hillingdon London Borough Council [2001] 2 AC 619 - example of assumption of
responsibility by individual psychologist to child.
X (minors) vBedfordshire County Council [1995] 2 AC 633 - authority can only act through its
employees.
(B) JUSTICIABILITY

Home Office vDorset Yacht Co Ltd [1970] AC 1004 - illustrations of justiciability; suggestion of ultra
vires test to define public authoritys liability.
X (minors) v Bedfordshire County Council [1995] 2 AC 633 - rejection of ultra vires test; ask instead
whether authority had acted within its discretion, and apply policy/operational distinction.
Barrett vEnfield London Borough Council [2001] 2 AC 550.
Phelps v Hillingdon London Borough Council [2000] 3 WLR 776
Carty v Croydon London Borough Council [2005] 1 WLR 2312, especially at [20]-[37] - rejection of the
discretion test; scepticism about the value of the policy/operational distinction.
Connor v Surrey County Council [2010] EWCA Civ 286 [76]-[102] (Laws LJ) - arguing that the
underlying idea of justiciability has changed
Craig, Administrative Law (L&O 504-505)
(C) ARE PUBLIC LAW CONCEPTS RELEVANT?
Stovin vWise [1996] AC 923 - dicta that claimant must show irrationality where duty based on failure to
exercise statutory power.
Gorringe v Calderdale Metropolitan Borough Council [2004] 1 WLR 1057 - some caution about the
dicta in Stovin. See in particular [4] (Lord Steyn), [26] and [31] (Lord Hoffmann). But contrast [91]
(Lord Rodger).

(2) Applying the test for a duty of care


Caparo Industries plc v Dickman [1990] 2 AC 605 - basic test for duty
X (minors) vBedfordshire County Council [1995] 2 AC 633 - not fair, just and reasonable to impose
liability for negligent conduct of child abuse investigations
Barrett vEnfield London Borough Council [2001] 2 AC 550 - fair just and reasonable to impose liability
for decisions as to childs future, once he had been taken into care.
Phelps vHillingdon London Borough Council [2000] 3 WLR 776 - assumption of responsibility by
psychologist employed by authority to child in respect of assessing child for dyslexia.
Stovin vWise [1996] AC 923 - vital importance of statutory context.
D v East Berkshire Community Health NHS Trust [2004] QB 558 (CA); [2005] 2 AC 373 (HL) - very
important case; you need to be familiar BOTH with the Court of Appeals analysis of the claim by the
child (which was not appealed to the House of Lords) AND with the House of Lords analysis for the
claim by the parents.

D v Bury Metropolitan Council [2006] 1 WLR 917, [1]-[32] only - wide application of D v East
Berkshire.
Jain v Trent Strategic Health Authority [2009] 1 AC 853
Merthyr Tydfil County Borough Council v C [2010] EWHC 62 (QB)

Seminar 6: Breach of duty and causation 1


(1) Breach of duty
(A) BASIC TEST
Whether D was negligent.
Blyth v Birmingham Waterworks Co (1856) 11 Ex 781: Negligence is the omission to do something
which a reasonable man, guided upon those considerations which ordinarily regulate the conduct of
human affairs, would do, or doing something which a prudent and reasonable man would not do.
(B) O BJECTIVE STANDARD
Objective standard;
Nettleship v Weston [1971] 2 QB 69 A learner driver crashed into a lamp post and injured her instructor.
CA said that negligence was judged by reference to an objective standard and in general, the reasonable
person test shouldn't be modified by reference to Ds characteristics.
To be judged in context;
This is particularly relevant to sporting situations (or situations analogous to sporting situations)
Wooldridge v Summer [1963] CA
Facts: A photographer is injured by one of the competitors horses while taking pictures at an show
jumping event.
Judgment: the CA held that the duty of care would only be breached where a competitor displayed
reckless disregard for the safety of the rider. So itd be necessary to show a high degree of negligence,
even recklessness. Here, D was held to have made an error of judgment rather than actionable
negligence.
Blake v Galloway [2004] (CA)
Facts: Teenage boys were playing a game which involved throwing pieces of bark at each other. One of
them threw a bit of bark so hard that it hit his friend in the eye, causing serious injury.

Judgment: CA said it was akin to a sport and there was an almost tacit agreement between the players as
to what theyd do. Thus the context allows the standard of the duty owed to be lowered and prevented the
finding of a breach.
Without reference to hindsight
You have to assess whether the defendant is careless by reference to the date of the event.
Roe v Minister of health [1954] CA
Facts: D ran a hospital. C had been in for an operation which required anaesthetic. He was injected with
anaesthetic but unfortunately, itd become contaminated and the contamination in it caused C to become
paralysed. The contamination had come about in a peculiar way. Anaesthetic was stored in glass vessels
called vials. These vials were stored in an antiseptic called phenol. Nobody realised it at the time but
those glass vials were at risk of microscopic cracking, through which the antiseptic could penetrate. It was
this penetration that had caused the contamination in Roes case.
Roes case was the first time anybody realised that this was how anaesthetic could become contaminated.
And after it was realised, articles were published about this risk etc Roe sues the hospital for
negligence.
Judgment: CA says hospital wasnt negligent as it has to be judged by the state of expertise as it exists at
the time of the alleged negligence. You cant use the benefit of hindsight. (Per Denning we must not
look at the 1947 accident with 1954 spectacles)

So we start from a basic position of an objective standard just by reference to the context and the time of
the incident. There are 2 questions we might ask:
-

Are there any characteristics of the particular defendant that can be taken into account?
What are certain factors the RP would consider in deciding whether to do (or not do) an activity?

BUT COURTS SOMETIMES TAKE DS SPECIAL CHARACTERISTICS INTO ACCOUNT:


Age, specifically young age
Mullin v Richards [1998] 1 WLR 1304
Facts: Two 15-year old schoolgirls were fencing with plastic rulers during a class when one of the rulers
snapped and a fragment of plastic caused one of them to go blind in one eye.
Judgment: CA held that the standard of care should be modified to the extent of taking Ds age into
account as people who are young are less capable of appreciating the likely consequences of their action.
Notes: From this judgment, is not clear whether either old age or mental immaturity can be taken into
account. (e.g. 20 year old with a mental age of 10)
Professional Status

Phillips v Williams Whiteley Ltd [1938] 1 All ER 566


Facts: C had her ears pierced by a jeweller in a department store. He sterilised the needle in a flame as
was the practice for jewellers then but did not take the steps a doctor would have. C gets an infection.
Judgment: CA held that it is the nature of the task that determines the standard of care. Also, where a
person undertakes an activity requiring specialist skills, theyre required to reachthe standard of a person
reasonably competent in that skill.
Disability
Mansfield v Weetabix Ltd [1998] 1 WLR 1263
Facts: Driver of a Weetabix lorry suffering from malignant insulinoma crashed into the front of a shop.
The disease makes you drowsy and affects your concentration and depth perception. Also, it comes on
gradually so it isnt easy to appreciate that youve been affected by it. Importantly, the driver didnt
realise he was suffering from it as for those who do, treatment is available.
Judgment: CA said to disregard the disability would essentially be to impose strict liability which is not
what the law of negligence is about. The RP test can thus be modified to take disability into account.
(Here, a reasonable person who didnt know he suffered from MI, and thus wouldn't have realised
something was wrong, would have continued to drive)
CERTAIN FACTORS A REASONABLE PERSON TAKES INTO ACCOUNT WHEN DECIDING WHETHER TO
PERFORM (OR NOT PERFORM ) AN ACTIVITY
1.
2.
3.
4.

Likelihood of Harm
Gravity of the Harm
Cost of taking precautions
Purpose

Likelihood of harm
Bolton v Stone [1951] AC 850
Facts: C lives close to a cricket ground and is hit by a cricket ball which is hit out of the ground. In this
ground, there was a fence around the edge which was about 7 feet high and the ground was on a bit of a
slope. In fact, for the ball to hit the claimant, it had to have travelled about 17 feet high in the air. And in
the past 30 years it was said, the ball had only been hit out of the ground on half a dozen or so occasions,
and prior to this, no one had been injured.
Judgment: The court held that low likelihood of harm meant it was not negligent for the club to have a
match there. So the fact that the RP would only have expected this to happen in a fantastical combination
of circumstances went to show there wasnt a breach of doc
This was confirmed more recently in
Whippey v Jones [2009] EWCA Civ 452

Facts: C was a runner in a park. D was owner of hector, the Great Dane who apparently weighed 12 stone.
Unfortunately for C, Hector knocked into his right shoulder and through a series of events caused him to
break his ankle. C sues D for negligence.
Judgment: CA holds its not negligent to let dog off leash in the park because dog never had previous
history of jumping onto people or barging into them. I.e. because likelihood of harm is so small, a RP is
justified in taking the risk of that harm
[16] Before holding that a person's standard of care has fallen below the objective standard expected
and so finding that he acted negligently, the court must be satisfied that a reasonable person in the
position of the defendant (i.e. the person who caused the incident) would contemplate that injury is likely
to follow from his acts or omissions. Nor is the remote possibility of injury enough; there must be a
sufficient probability of injury to lead a reasonable person (in the position of the defendant) to anticipate
it.
Gravity of Harm
Paris v Stepney Borough Council [1951] AC 367
Facts: C worked for the council on a street cleaning vehicle. He only had one functioning eye. When
using a hammer to remove a bolt on a vehicle, a scrap of metal is liberated by this act and it goes into his
good eye. So he completely loses his sight as a result of the accident. He sues the council saying theyre
negligent in failing to provide him with goggles in the use of the vehicles
Judgment: D owed a higher standard of care to the claimant because they knew that for him in particular,
an injury to his good eye would be extremely serious.
Cost (and practicability) of precautions
Latimer v AEC Ltd [1953] AC 643
Facts: Here, thered been a downfall of rain which had caused flooding in Ds factory. As a result of that
flooding, coolant for the machines which ran in open channels on the floor had been washed out of those
channels. After the water subsided, that coolant which was like oil had been left all over the factory floor.
D realised there was a problem and they obtained 3 tonnes of sawdust to sprinkle over the floor to make it
less slippery, they then reopened the factory.
C was a worker who slipped and fell on a staircase on a set of stairs that didnt have this sawdust (but had
coolant). It was a set of stairs that was rarely used.
Judgment: HL said one had to take into account the economic cost of closing the factory for a night in
deciding whether employees had taken reasonable care in reopening it after taking the precautions they
had taken. The only way to remove the risk would have been to close the affected part of the factory until
it dried out. This would have been expensive and disproportionate to the relatively small risk of injury.
Purpose
Watt v Hertfordshire CC [1954] 1 WLR 835

Facts: C was a fireman. He was injured whilst travelling to an emergency call. The circumstances of that
emergency were slightly peculiar. Fire brigade has received this call in relation to a person trapped in a
vehicle after a road accident.
For some reason, the fire brigade didnt have the correct vehicle for transporting the equipment theyd
need to extract the person from the road accident. As such, they put it on a different vehicle not designed
to carry it. C is on back of vehicle, basically holding the equipment in place. When the vehicle comes to a
stop, the equipment moves on the back of the vehicles and crushes/injures C
Judgment: CA said theres no breach of duty of care as the purpose for which the risk has been taken was
to get to the scene ASAP to help the trapped person. So that purpose, that socially beneficial purpose (to
save a life), altered the assessment of whether the D, the fire brigade, had been negligent.
Notes: Even a socially beneficial purpose does not mean the defendant is justified in taking any risk.
Emergency services for example, must still take care in passing re lights and remember to use their sirens
and lights. Also now, s1 of the Compensation Act 2006 allows the court to consider whether
precautionary or defensive measures might prevent a socially desirable activity.
(C) LEARNED HAND FORMULA
There is a breach of duty if the cost of precaution is less than likelihood of the harm multiplied by the
gravity of the harm
C<L*G
By doing this, American courts have felt they give more certainty to the way the breach of duty
assessment is carried out. There are potential problems though
(D) SPECIFIC ISSUES REGARDING BREACH OF DUTY CONCERNING PROFESSIONALS
Bolam v Friern Hospital Management Committee [1957] 1 WLR 582
Facts: Person treated for depression by electric shock therapy. Consequence of being administered with
the treatment is that your limbs flair around due to the force of the electric shock. On the facts of this
case, the patient had been strapped onto the bed in order to stop him falling off when the electricity had
been administered. Unfortunately hed been strapped on so tightly and reacted extremely forcibly so that
when he did move, he broke his leg. He argued itd been negligent not to give him any muscle relaxing
drug before administering the therapy. However, there was some issue about whether they should have
administered it
Judgment: J McNair resolved the problem by saying it wasnt necessary to show that what a particular
doctor had done in the circumstances was what reasonable doctors would do in the case. So long as what
was done conformed with a responsible body of medical opinion, itd satisfy the test for taking reasonable
care.
Its been criticised for being too protective of professionals. The HL clarified the situation;
Bolitho v City and Hackney HA [1998] AC 232

Facts: C suffered brain damage as a result of a doctors failure to attend to clear a childs blocked airways
by intubation. There was a difference of medical opinion as to whether intubation was necessary in the
particular circumstances
Judgment: Its necessary to satisfy the test to show that the responsible body of medical opinion you were
following was both based on somerational foundation and that it had made an assessment of both the
advantages and disadvantages of pursuing that particular approach or course of treatment.
Though the tests from Bolam and Bolitho are primarily developed for doctors, they apply to all
professions.
SUMARRY
-

You typically have a breach of duty where D has been negligent. Per Blyth v Birmingham
Waterworks negligence is doing something a RP person wouldnt or not doing something a
reasonable person would.
When we talk about the RP here, we mean it is judged from an objective standard (Nettleship v
Weston) taking into account the context (Wooldrige v Summer, Blake v Galloway), and without the
benefit of hindsight (Roe v Minister of Health)
However, certain characteristics of the particular D can be taken into account. Notably:
O Age, especially young age (Mullins v Richards)
O Profession (Phillips)
O Disability (Weetabix)
When considering the factors a RP would take into account, there are 4 main things we look at
O Likelihood of harm occurring (Bolton v Stone, Whippey v Jones)
O Gravity of the harm (Paris)
O Cost and practicability of precautions (Latimer v AEC)
O Purpose (so socially desirable purposes will have a lower standard of duty owed to C
Watt; nb. Compensation Act 2006 s1)
Also take note of the learned hand formula

(2) Causation 1
(A) THE BASIC BUT FOR TEST
Starting point for causation is the basic but for test. It works on the balance of probabilities.
Barnett v Chelsea & Kensington Hospital Management Committee [1969] 1 QB 428
Facts: C was turned away from the hospital by a doctor who refused to examine him. He later died of
arsenic poisoning. It was later shown that he would not have recovered even if properly diagnosed as it
was too late to administer the antidote at that point.
Judgment: No liability for breach of duty as there was no causation
(B) EXCEPTION : MATERIAL CONTRIBUTION TO DAMAGE

Where there are multiple causes of damage, It can be hard to prove but for causation. Thus, the claimant
only has to show that D made a material contribution to damage)
Bonnington Castings Ltd v Wardlaw [1956] AC 613
Facts: C contracted pneumoconiosis while comes about through breathing contaminated air. (It was
contaminated with silicone dust in this case) There were two main causes of dust, one of which was
required by law to be extracted (and so, only one cause was negligent) It was impossible to prove which
dust had caused the disease.
Judgment: D could be liable as they were responsible for a material contribution to the damage. It does
not have to be a significant cause but it has to be a material contribution rather than a negligible one. (Per
Lord Reed: if you were responsible for a single speck of dust, you wouldnt be liable)
(C) EXCEPTION : MATERIAL CONTRIBUTION TO RISK OF DAMAGE
McGhee v National Coal Board [1973] 1 WLR 1
Facts: Here, C gets dermatitis which he alleges is a result of exposure to brick dust while cleaning out
brick kilts. While D is not negligent in respect of that exposure, it is alleged that they are negligent in
failing to provide washing facilities. As it were, C had to cycle home covered in the dust.
The problem here is that no one is quite sure exactly how the dermatitis comes about. If it occurs the
moment the dust lands, say it causes an abrasion of the skin which will inevitably result in dermatitis,
since D is not negligent at that point, it seems thered be no liability.
However, if it is caused through prolonged contact with the skin, cycling home with the dust has clearly
contributed to the disease occurring. The point here is that we really do not know. The sole causal link
that C can establish is that the employers action (or inaction rather) increased the risk of getting
dermatitis.
Judgment: HL here held that a material increase in the risk of damage was sufficient for liability. This was
quite a radical decision and in Wilsher, they attempted to re-interpret McGhee to the point where it would
be as though it never happened.
Wilsher v Essex Area Health Authority [1988] AC 1074
Facts: The defendant hospital, initially acting through an inexperienced junior doctor, negligently
administered excessive oxygen during the post-natal care of a premature child who subsequently became
blind. Excessive oxygen was, according to the medical evidence, one of five possible factors that could
have led to blindness. On the "balance of probabilities" test, the hospital would not be liable, since it was
more likely that one of the alternate risks had caused the injury. (CA found liability applying McGhee)
Judgment: The House of Lords found that it was impossible to say that the defendant's negligence had
caused, or materially contributed, to the injury and the claim was dismissed. It also stated that McGhee
articulated no new rule of law, but was rather based upon a robust inference of fact
This reinterpretation however was subsequently rejected.

Fairchild v Glenhaven Funeral Services Ltd [2003] 1 AC 32


Facts: C had worked for several different employers, all of whom had exposed him to asbestos. The (late)
claimant contracted mesothelioma (a form of cancer that is almost always caused by exposure to asbestos)
and died. His wife sued the employers on his behalf in negligence. However, due to long latency periods
(it takes years before symptoms are evident), and the fact that a single fibre of asbestos can trigger
mesothelioma, it was impossible to attribute his death to a single employer generally, nor on the balance
of probabilities.
Judgment: The HL reaffirms the McGhee test of material contribution to the risk of harm and rejects the
Wilsher analysis (but approves the result) and thus, all the employers were jointly and severally liable.
Notes: So we have this new principle based on increase in risk but we know that in Wilsher there was no
liability but in McGhee there was. So were left with the question of when an increase in risk will be
sufficient.These limits on the Fairchild exception come through quite clearly in the Barker case
Barker v Corus UK Ltd [2006] 2 AC 572
Facts: Cs husband, who died of mesothelioma, had been exposed to asbestos during three periods in his
working life: first while working for a company which had since become insolvent, secondly while
working for D and thirdly, while self-employed. (at first instance, D and the insolvent company were held
to be jointly and severally liable subject to a 20% reduction for contributory negligence)
Judgment: The HL said it didnt matter that not all of the exposures were wrongly. Therefore, a defendant
who wrongly exposed the deceased to the risk may still be liable even though the other exposures either
occurred naturally or resulted from the deceaseds own acts.Also, they applied proportionate liability;
Notes: This clarification leaves us with 2 limits to applying Fairchild/Barker.
1. Scientific uncertainty The idea that it is the limits of science which prevents the identification of
the actual defendant responsible for the damage and that this limit shouldnt bar C from a claim.
2. Same causal agent/mechanism Since Wilsher was upheld, it seems that to apply Fairchild, the
damage must have the same causal agent (see notes on the dicta explaining this)
On proportionate liability, Lord Hoffman and the majority thought it was more fair. Lord Rodger in
dissenting felt that to apply proportionate liability was to make Fairchild not an exception to but for
causation but a new method of causation in itself.
For various reasons, this caused a political backlash resulting in s3 of the Compensation Act 2006 which
specifically only refers to cases of mesothelioma caused by exposure to asbestos.
SUMMARY
-

Starting point is but for causation (Barnett)


There is an exception where D was responsible for a material contribution to the damage
(Bonnington)
There is a further exception where D is responsible for a material increase in risk of damage
(Barker but also important are the cases leading up to Barker)

McGhee material increase in risk of damage was sufficient for liability


Wilsher material increase of risk was not sufficient. McGhee reinterpreted
Fairchild Essentially follows McGhee and says that a material increase in risk of
damage was sufficient and while they reject the Wilsher analysis, they approve the result
Barker resolves the tension between Fairchild and Mcghee somewhat
o Says that D can be liable when responsible for a material risk in increase of damage
regardless of whether all the sources of exposure were negligent
o However, they favoured proportionate liability which per Lord Rodgers reasoning
seems to make this not an exception to but for causation but a new form of causation
o Importantly, to use the Fairchild exception, it seems you need
Scientific uncertainty
Same causal agent/mechanism
o
o
o

Seminar 7: Causation 2 and remoteness


(1) Causation 2
(A) LOSS OF A CHANCE
Courts are extremely reluctant to impose liability where the negligence of the defendant caused the
claimant to lose a chance
Hotson v East Berkshire Area Health Authority [1987] AC 750
Facts: a boy fractured his hip when he fell from a tree. The hospital made a misdiagnosis and the boy
developed a hip deformity. Experts confirmed that he would have had a 75% chance of developing the
deformity even with the correct diagnosis. The trial judge (upheld by CA) awarded him 25% in damages
for his lost chance of recovery.
Judgment: HL reversed the previous decisions considering that since there was only a 25% chance that
the negligence caused the injury, but for causation was not satisfied.
Gregg v Scott [2005] 2 AC 176
Facts: D had negligently misdiagnosed the claimants malignant cancer as benign. This delayed the
claimants treatment by nine months and reduced his chances of being cured from 42% to 25%.
Judgement: The majority of the HL (3-2) upheld the earlier decision in Hotson, though Lord Nicholls
(joined by Lord Hope) argued strongly for loss of a chance to be actionable.
Notes: If claims for a loss of chance were actionable, it is likely there would be pressure for all damages
to be awarded on proportionate liability. Also theres the floodgates argument (people made even a few %
worse off by medical negligence might try and claim)Finally, statistics arent perfect but with BoP, at least
you have a higher chance of getting it right.
(B) I NTERVENING ACTS

An intervening act may break the chain of causation between the defendants breach of duty and the loss
or damage suffered by the claimant.
In Environment Agency (formerly National Rivers Authority) v. Empress Car Co. (Abertillery) Ltd .
(1998) 2 WLR. 350, Lord Hoffman says there are 2 strong indicators to look out for
-

Deliberate voluntary acts


Extraordinary natural events

The third type is a bit more tricky, namely, negligent acts


Extraordinary natural events
Hoffmann illustrates this in the case. Imagine Ive got a drum full of flammable liquid on my premises. If
that liquid catches fire because its a very warm day and that makes it spontaneously combust etc, we
wouldnt regard it as something that breaks the chain. In the scheme of things, it being a hot day isnt
particularly unusual.
If on the other hand, the drum is struck by lightning during a storm and that causes it to catch fire, that
would be an extraordinary natural event.
Deliberate voluntary acts
Similarly in relation to deliberate voluntary acts, talking about the same drum, theres a workman
smoking who casually negligently drops the cigarette into the drum that probably wouldnt break the
chain of causation.
But if he deliberately puts it in knowing theres flammable liquid in the drum that probably would break
the chain of causation.
Negligent acts
The other thing that might break a chain of causation would be a negligent act. However, this is going to
be more difficult to establish.
(Phrased positively, it seems the original defendant will be responsible for injury and damage which are
the natural and probable results of the initial wrongful act; he will also be liable where the intervening act
is one he should have foreseen)
(phrased negatively, there seem to be three main things against the negligent act breaking the chain (i)
where the nature of the duty is such that it will be made redundant by breaking the chian, (ii) where the
negligent act is quite unusual or extraordinary, (iii) where the claimant himself breaks the chain by doing
something stupidly negligent)
The final point Hoffman made in Empress Car is about the relation between the rules on intervening acts
and the rules of a DoC to prevent damage being inflicted by a TP (e.g. smith v littlewoods). (This point is
also made in Reeves) is that the court has to consider the nature of the DoC.
Reeves v Metropolitan Police Commissioner [2000] 1 AC 360

Facts: The deceased (C) was in the prison and committed suicide (after 2 earlier failed attempts). The
police knew he was a suicide risk. It was held that the police did owe him a DoC to protect C from
injuring himself which they breached by leaving the flap of the cell door open. (remember the rules on
DoC for TPs)
Judgment: Applying the general principles about intervening acts, it seems like the chain of causation is
broken by a voluntary deliberate act. However, the courts decided that the intervening act principle
couldnt be established.
Notes: Here, the point of the duty is to prevent a person from deliberately harming himself. If at the
causation stage of the claim the court were to say that hes broken the chain by deliberately harming
himself that would be making a mockery of the duty.
So if a duty is express in terms of being a duty to prevent either someone injuring himself or a duty to
prevent another person injuring him, then these general rules about breaking the chain of causation dont
apply. The chain will not be broken by voluntary deliberate acts by C or a TP.
Knightley v Johns [1982] 1 WLR 349
Facts: Theres a car accident in the Queensway tunnel in Birmingham. D1 is the person who negligently
crashed his car. Police come and they realise they havent stopped incoming traffic from coming in. So
they send 2 cops back into the tunnel into oncoming traffic (on bike) and a car comes and crashes into one
of bikers.
The police officer is clearly negligent or at least in contravention of police standing orders. First of all
they failed to stop the traffic at the correct end of the tunnel. Then the officer in charge of the incident
orders the bikers to go the wrong way and the police biker is suing the original accident causer.
Judgment: The CA is quite clear that the mere fact that an intervening act is negligent does not break the
chain of causation (i.e. negligent acts do not automatically break the chain of causation); the negligent act
needs to be quite unusual or extraordinary. Here, the CA emphasised that the two deliberate
contraventions of police orders, namely blocking the wrong end and then sending the officers against the
flow of traffic, were unusual acts of negligence and thus were held to break the chain of causation. (The
original defendant will be responsible for injury and damage which are the natural and probably results
of the [initial] wrongful act)
Further, it is also possible for the claimant to break the chain of causation by doing something stupidly
negligent. (Highlighted in Spencer). More specifically, for an act of a claimant to be a novus actus
interveniens, it must be entirely unreasonable in all the circumstances.
Spencer v Wincanton Holdings Ltd [2010] PIQR P8
Facts: C suffered injury due to workplace negligence and had to have an amputation of one leg (or most
of it). While he had prosthetics, pending modification to his car he could not drive it with the prosthetics.
3 years after the injury while filling his car at a gas station, without wearing prosthetics or using crutches,
he tripped and suffered injury which made him wheelchair bound. He sued D in respect of the further

damage he suffered. At first instance the judge held that the chain of causation was not broken but
reduced damage by a third for contributory negligence.
Judgment:
The second ground of appeal based on the respondents unreasonable conduct also failed. There was no
novus actus interveniens that broke the chain of causation. Whilst each case must be considered on its
own facts the trial judge was entitled to find that the respondents conduct was not unreasonable as
contended for. The respondent made a misjudgement in running a risk by not using his prosthesis or sticks
and this was properly reflected in the reduction for contributory negligence.
(2) REMOTENESS
The Wagon Mound (No 1) [1961] AC 388
Facts: Overseas Tankship had a ship, the Wagon Mound, docked in Sydney Harbour in October 1951. The
crew had carelessly allowed furnace oil (also referred to as bunkering oil) to leak from their ship. The oil
drifted under a wharf thickly coating the water and the shore where other ships were being repaired. Hot
metal produced by welders using oxyacetylene torches on the respondent's timber wharf (Mort's Dock) at
Sheerlegs Wharf fell on floating cotton waste which ignited the oil on the water. The wharf and ship
moored there sustained substantial fire damage. In an action by Mort's Dock for damages for negligence it
was found as a fact that the defendants did not know and could not reasonably have been expected to
know that the oil was capable of being set alight when spread on water. The dock owners knew the oil
was there, and continued to use welders.
Judgment: Here the PC decided that the main link between damage and liability is foreseeability and that
it is not only that damage generally must be foreseeable but the type of damage as well.
Here, the fire damage was not foreseeable and thus D was not liable. (Damage generally could be
foreseen though. For example, environmental damage from the oil potentially leading to property
damage. Under the only rule from Re Polemis, this would have sufficed.)
It is sufficient to foresee the type of harm, not the magnitude (Essentially the egg-shell skull rule)
Smith v Leech Brain & Co [1962] 2 QB 405
Facts: C was splashed by molten metal as a result of his employers negligence and suffered a burn to his
lip. This burn triggered cancer, from which the claimant died. The claimants lip was pre-malignant at the
time of the incident.
Judgment: There is no dispute as to liability for the initial injury. Further, the HL says that while the
subsequent cancer was not foreseeable, it is merely the extent of the injury and the remoteness rules
merely require you to foresee the type of damage. (sometimes called the eggshell skull rule; per Lord
Parker If a man is negligently run over... it is no answer to the sufferers claim for damages that he
would have suffered less injury... if he had not had an unusually thin skull or an unusually weak heart)
It is sufficient that you foresee the type of harm may come about and not the exact method it comes
about

(e.g. in wagon mound theres a distinction between damage caused merely by oil and that by fire)
Hughes v Lord Advocate [1963] AC 837
Facts:On November 8, 1958 evening the appellant, an eight year old boy with his ten year old uncle was
walking down Russell Road, Edinburgh. Some Post Office employees were repairing cables under the
street. They opened a manhole on the surface of the road, which was nine feet deep and put a weather tent
on it. A ladder was put inside the manhole for access. The tent was again covered with a tarpaulin for
better protection, but the workmen left one of the corners a gap of two feet and six inch. They had also
fixed four red paraffin lamps on the site to warn the traffic since 3.30pm. The workmen left the site at
about 5pm for a tea break to a nearby Post Office building. Before leaving, they took out the ladder and
put it on the ground outside the tent.
While the workmen were out, the plaintiff and his uncle arrived at the site and started meddling with the
equipment. They picked up one of the lamps and entered the tent. They took the ladder along with which
was kept outside the site in order to explore the manhole. Thereafter, they took a piece of rope (which was
not a part of the Post Office equipment) and tied it to the lamp and went inside the manhole. After
exploring the manhole they succeeded to come out of the manhole safely. Somehow, the appellant tripped
over the lamp, and it fell into the manhole. The lamp broke, the paraffin within leaked, the paraffin
vaporised which resulted to an explosion with flames reaching up to thirty feet. Due to the impact of the
blast, the appellant fell into the hole and suffered severe injuries from burns.
Judgment: The HL held that knowledge of the source of danger would suffice. So while it may not be
foreseeable that a small boy would come along to the manhole with the lamp, pick it up, go in, come out,
and then drop the lamp causing an explosion causing him to fall back in and get hurt, you could probably
foresee that a child may come along and get hurt.
So it is important to define the kinds of foreseeable damage by reference to the result and what might be
termed the basic method by which the result comes out. So in Hughes, the type of damage is personal
injury by getting burnt and it doesnt matter that the detailed way in which the harm came about was
unforeseeable. So both the extent of the harm and the detailed method by which it came about neednt be
foreseeable as long as the type of harm is foreseeably and it that it was reasonably foreseeable that the
source of danger might cause that type of danger.
What counts as a different type of harm, and what merely goes to the extent of the same type of harm?
(Courts seem to take a wide interpretation of the extent)
(focus on Page v Smith and Corr v IBC which shed some light on this question)
Corr v IBC [2008] UKHL 13
Facts:As a result of Ds negligence C suffers a severe head injury at work. He receives surgery but begins
to suffer from PTSD and clinical depression as a result of the injury and a few years later, kills himself.
Judgment:HL say the suicide is effectively a kind of symptom/a facet of the clinical depression. Its
merely the extent of the harm, not a different type of the harm. So it wasnt necessary to foresee hed

commit suicide as a result of getting this type of head injury. All it was necessary to foresee is that hed
suffer depression and the suicide goes to the extent of that depression
Notes: If all that needs to be foreseen is that the harm may be suffered in a broad sense, that doesnt really
insist on a tight test. This is the reason courts have been focusing on this point.
Page v Smith [1996] AC 155
Facts:C involved in a minor car accident and was unjury but this caused a recurrence of his ME
Judgment: In the case of direct victims, their Lordships said the following test should be applied: "Could
the defendant reasonably foresee that his conduct would expose the plaintiff to the risk of personal injury,
psychological or physical?" If the answer was yes, it would be irrelevant that the extent of the damage
was unforeseeable because the plaintiff had special sensitivities. This is based on the eggshell skull rule,
that is, one "takes the plaintiff as one finds him". Consequently, the defendant was found liable for the
nervous shock suffered by Mr Page.

Summary
-

Loss of a chance is not sufficient to warrant an actionable claim (Hotson)


Confirmed in Gregg but with a particularly strong dissent this time. There are legitimate concerns
against allowing it though
o Might mean a push for all claims to be based on proportionate liability
o Floodgates
o Statistics arent perfect but at least with BoP you have a greater chance of getting it right

Seminar 8: Defences to Negligence Claims


(1) Voluntary assumption of risk (volenti non fit injuria = no wrong is done to
one who consents)
NB this defence should not be confused with the argument that can be made in response to battery claims
(discussed in seminar 12) that the claimant has consented to the defendants touching; here the point is
that the defendant has voluntarily assumed the risk of harm, not that he has consented to the harm itself.
The most common situation is where it can be concluded from the factual situation that C has consented
to the risk
Woodley v Metropolitan District Railway Co (1877) 2 Ex D 384
The plaintiff, a workman in the employ of a contractor engaged by the defendants, had to work in a dark
tunnel rendered dangerous by the passing of trains. After he had been working a fortnight he was injured
by a passing train. The jury found that the defendants in not adopting any precautions for the protection of
the plaintiff had been guilty of negligence. (CA reversed this but HL returned to the original verdict)

Essentially, one mustnt confuse knowledge and consent of the risk. It is not enough that C knew that the
risk was there: one must prove consent to that risk
Dann v Hamilton [1939] 1 KB 509
Facts: On a motoring trip, participants go to various pubs on the way to London and back out. This affects
drivers ability to drive. During the trip, some passengers become so alarmed by the way that the driver is
driving that they abandon the car. Car crashes, driver dies and C is badly injured.
The defences claimed are both (1) Contributory negligence (put aside for now) and (2) volenti non fit
injuria
Judgment: The defence failed: Held that it only really applies in extreme cases. There would be 2
situations in which the defence of volenti might be appropriate1. Implied permission to be careless- C gave permission to D to act carelessly
2. Deciding to enter an obviously dangerous situation- e.g. meddling with an unexploded bomb
Notes: Getting into a car with an obviously drunken driver does seem like deciding to enter into an
obviously dangerous situation. The defence probably failed for historical reasons. This case was in 1939
and drink driving was still very common and not really frowned upon. So it may well be that in its
context, this case makes sense: because at the time the obviousness of the danger isnt as glaring as it
would be to us today
A. I MPLIED PERMISSION TO BE CARELESS - C GAVE PERMISSION TO D TO ACT CARELESSLY
ICI Ltd v Shatwell [1965] AC 656
Facts: C works in a quarry as a shot-firer. His job was to blast away rocks that were used for industrial
purposes. Before they carried out the blast, they were instructed to run a test in which everyone else had
to be well away from the area in which the explosive was to go off: to carry this out, he had to use long
wires to operate the explosive from the shelter in which they were situated at.
On the day of the test, they didnt have the long wires. While one person went off to get the long wires,
the other two decided to carry out the test with short wires. The bomb explodes and the C is injured. He
seeks to sue his employer for vicarious liability for the carelessness of his work colleague
Judgment: the defence is established. HL emphasises that all the instruction and training that had been
given to the shot firers, and that previously one of the shot firers had been fired for not carrying out the
tests. Given that factual background, the actions of the 2 shot firers in testing the circuit using the short
wire gave rise to an implied promise by the man who was injured not to sue the person who injured him.
Notes: This almost seems to draw on contractual ideas
B. DECIDING TO ENTER AN OBVIOUSLY DANGEROUS SITUATION
Morris v Murray [1991] 2 QB 6

Facts: C and D are friends, and had been out drinking heavily. C drives them from the pub to the airport.
At the airport, they take off the wrong way down the runway and fly off with the wind behind them
(should have taken off into the wind). The plane crashes and the pilot dies, with C severely injured. Has
volenti been established here? (following Dann v Hamilton, it seems that it shouldnt be established)
Judgement: The court applied a subjective test and held that the claimant was aware of the risk he was
taking and therefore consent was found (in contrast with Dan)
Notes: Though they took a subjective approach, the CA was slightly troubled by the argument that if C
was so drunk as to not appreciate the risk, consent wouldnt be found. So in a sense, his legal problem
was that he was not drunk enough. If a more contract like theory is taken, an objective approach would
be taken. Mitchell feels the subjective approach is appropriate here though.
Woodridge v Sumner [1963] 2 QB 23: (About the photographer injured at a horse race)
Notes: CA said that the volentidefence isnt very useful in large group situations. They feel it is far better
to deal with these situations by adjusting the standard of care that was owed. Essentially, not that C
agreed to D being careless but that C wasnt entitled to demand D reach the level of care that he would
ordinarily have to reach.
So in these situations, they can be dealt with either by the volentidefence or by changing the standard or
care to reflect what we think C should be entitled to demand in the circumstances
Corr v IBC Vehicles Ltd [2008] AC 884(negligent injury leading to subsequent depression and suicide)
Notes: Here it was argued that in deciding to jump off from a height, C entered into an obviously
dangerous situation. Nonetheless, the defence of volenti failed. Despite features that seem to make it a
promising argument. It was held that the depression had so affected his mental processes that he couldnt
evaluate the merits of not jumping off the car park. There was therefore no consent to a risk, because there
was no true/ free consent. (Again, this highlights that the test is objective)

(2) Contributory negligence


Law Reform (Contributory Negligence) Act 1945, s 1(1): Where any person suffers damage as the
result partly of his own fault and partly of the fault of any other person or persons, a claim in respect of
that damage shall not be defeated by reason of the fault of the person suffering the damage, but the
damage recoverable in respect thereof shall be reduced to such extent as the court thinks just and
equitable having regard to the claimants share in the responsibility for the damage.
Fault is defined in s 4 to mean negligence, breach of statutory duty or other act or omission which gives
rise to a liability in tort or would, apart from this Act, give rise to the defence of contributory negligence.
Note: The definition of fault in the act is important. Pre-1945, the defence was only applicable to certain
torts. Post-1945, courts can apply it to the extent it thinks its applicable. The Act only reforms the effect
of contributory negligence though, it doesnt reform when the defence is available nor does it expand the
categories.

WHERE ANY PERSON SUFFERS DAMAGE AS THE RESULT PARTLY OF HIS OWN FAULT AND PARTLY
OF THE FAULT OF ANY OTHER PERSON OR PERSONS S O BOTH PARTIES MUST HAVE BEEN AT
FAULT

Gough v Thorne [1966] 1 WLR 1387


Facts: 7 year old girl is crossing the road with her mum and a lorry stops to let them through. Once they
pass the lorry, they get hit by a car that was trying to overtake the lorry. Had the little girl been negligent
in crossing the road without checking and relying simply on the indication by the lorry driver and the
elder sibling assisting her?
Judgment: It was reasonable reliance- the test of fault does seem totake into account the same factors we
take into account in establishing that D were in breach of duty. Should there be a more complicated
situation with more factors, we might look at how C weighed up these factors.
Jones v Livox Quarries Ltd [1952] 2 QB 608
Causation is somewhat important to contributory negligence. So Cs fault has to have caused some part of
the damage.
Facts: C was injured at work when two quarrying vehicles collided. The claimant was sat on the back of
one of the vehicles at the time of the collision, without the drivers knowledge and in contravention of the
explicit prohibition in doing so. Applying the last opportunity doctrine, one wouldnt say that Cs fault
caused his damage at all. Thus, CA had to consider if the doctrine still applied post-1945.
Judgment: Given that contributory negligence post 1945 is no longer a complete defence, the doctrine,
that was developed to deal with the harshness of the fact that it was a complete defence, doesnt need to
apply and Cs act was taken to be contributory negligence. (recall again that finding fault under the act is
similar to finding a breach of duty. So assessed by reference to a reasonable person)
Per Denning: In order to illustrate this question of causation, I may say that if the plaintiff, whilst he was
riding on the towbar, had been hit in the eye by a shot from a negligent sportsman, I should have thought
that the plaintiff's negligence would in no way be a cause of his injury. It would only be the circumstance
in which the cause operated. It would only be part of the history. But I cannot say that in the present case.
The man's negligence here was so much mixed up with his injury that it cannot be dismissed as mere
history. His dangerous position on the vehicle was one of the causes of his damage
Thus the approach towards causation here has become broader/more inclusive. Courts focus on whether
Cs fault is merely part of the history/background. If it is, it wont regard the fault as having caused the
loss because there wont be a sufficient connection in time, place and circumstances between Cs fault and
the damage that he suffers. (because above, Cs negligence in riding atop the excavator, even if shot,
would clearly satisfy but for causation)
BUT THE DAMAGE RECOVERABLE IN RESPECT THEREOF SHALL BE REDUCED TO SUCH EXTENT AS
THE COURT THINKS JUST AND EQUITABLE

The extent to which C owed actions that could have prevented the harms or reduced the severity: Froom
v Butcher [1976] QB 286: carelessly not wearing a seatbelt affects the severity of the injury that is
suffered.
Carelessness Rule: If both have contributed equally but one has been more careless, then the more
careless one should pay a bit more. There is a problem of working out how much of a reduction to make
for the less careless party though.
Denning in Froom: Introduces guidelines for what the figure should be- but these guidelines havent
really been followed
(3) Illegality (ex turpi causa non oritur actio = no action arises from a disgraceful cause)
(3) ILLEGALITY (EX TURPI CAUSA NON ORITUR ACTIO = NO ACTION ARISES FROM A DISGRACEFUL
CAUSE )
C could lose claim for damage in negligence because C was involved in an illegal activity at the time.
There must be a close connection between the injury sustained by C and the illegal activity for the
defence to work.
4 different approaches as to how the defence should work:
(1) The fact that C was doing a criminal offense should have no impact on the liability of D:Revill v
Newberry [1996] QB 567
Facts: C went to steal property from a shed but the owner was ready waiting with a shotgun. The owner
fired the gun in panic when C started to enter the shed causing serious injuries.
Judgment: CA held that the claimant should not be deprived of a claim on the basis of illegality, saying
that it was too far-reaching to deprive [the claimant] even of compensation for injury which he suffers
and which otherwise he is entitled to recover at law
(2) Cs illegality simply one factor that must be considered in the Caparo test under the heading of fair,
just and reasonable: Vellino v Chief Constable of Greater Manchester [2002] 1 WLR 218
This approach matters because if illegality is a defence in itself, the fact that C was involved in the illegal
conduct bars the claim- but if its just one factor in Caparo, it might be outweighed in regard to other
factors in determining if its FJR to impose a DOC
Facts: C a criminal, having a party in his flat when police arrest him. Tries to avoid arrest by jumping out
of his flat, gets injured in landing on the pavements. It was found that police didnt stop him from
jumping and stood to one side while he tried to jump. In seeking to escape from police custody, C was
committing a criminal offense. Thus police argued that no DoC owed because of this criminal act
Held: that no duty was indeed owed- but Sedley LJ dissented. He felt that Cs illegality was outweighed
by police failing to perform their duty to stop him jumping. Applying the Caparo test, the fact of Cs
illegality shouldnt be determinative because D had also committed a breach of duty.

(3) Illegality alters the standard of care that C is entitled to expect- it makes it impossible to ascertain
what that standard of care should be:Pitts v Hunt [1991] 1 QB 24
Facts: claim by a passenger on a motorcycle brought against the motorcycle driver. The driver had been
driving very recklessly with the encouragement of the passenger
Held: that in these circumstances, very hard to fix what the standard of care should be and what
reasonable care requires under those circumstances
(4) Claim should be barred where is one is relying on ones own criminal conduct: one shouldnt be
entitled to claim if ones negligence damages require one to assert that one had committed criminal
acts :Gray v Thames Trains Ltd [2009] 1 AC 1339
Facts:Claim from a victim of the Ladbroke Grove rail crash. Ds negligence caused him to suffer PTSDwhile suffering from PTSD he had stabbed and killed someone who had annoyed him. Thus at the time of
the trial, he was in a secure mental hospital. Claimed in respect of the PTSD suffered (D admitted
liability) and further claimed (Claims 1 and 2) for earnings that he had lost as a consequence of the PTSD
Claim 1: While in the mental hospital cannot earn any money at all (Complete loss of earnings)
Claim 2: As it is, his earning capacity with PTSD is lower than it would have been had been not injured
(Loss of earnings continuing into the future)
Makes no reference to the fact that he is now in a mental hospital
Held:
WRT Claim 1: Failed, because one cannot claim for lost earnings while in custody- one cannot claim for
the consequences of a criminal penalty that was imposed by court. It wouldnt be reflective of a consistent
legal system if the criminal system said that one had to go to a mental hospital, and another said that you
could get compensation for the penal sanction
WRT Claim 2: One cannot disregard the fact that he is now in mental hospital in a decision that he cannot
earn money. Failed again because court held that they couldnt ignore what the criminal court had donewouldnt assess damages pretending that C was still at liberty and all that was the issue was a reduction in
earning power. Effectively, would take into account the imposition of a criminal sentence because it did
cut off the Ds liability.
The vicissitudes of life can have the potential to affect life- at this point, Ds liability should be stopped
This doesnt mean that criminal sentences always stop tort rights rather, on the facts of this case, he
cannot get compensation
Impact of remoteness: Loss of earnings is classically foreseeable, and it doesnt matter that it came about
in a slightly strange way (via PTSD)
Check the HL arguments against claim 2- they seem quite silly
Summary

3 main defences
o Consent (Volenti)
o Contributory negligence
o Illegality (some dispute as to whether its a defence)
Consent
o Easiest way is to see if theres some express consent. Consent to the risk is not the same
as knowledge of the risk (Woodley v metropolitan district railway)
o Where the consent is not express, courts have indicated there are 2 main situations in
which it can be inferred (Dann)
Implied consent to the risk; (more) objective standard (Shatwell)
Deciding to enter an obviously dangerous situation (Morris)
Subjective test (Morris and Corr)
Also in situations witch large groups, particularly sporting events, it
might make more sense to lower the standard of care rather than
enquiring into the individuals state of mind (Wooldridge)
Contributory negligence
o Covered by the Law Reform (Contributory Negligence) Act 1945
S1 explains the workings
S4 defines fault (essentially uses breach of duty analysis)
o Pre-1945, contributory negligence was a complete defence. This was somewhat mitigated
by the last opportunity doctrine.
o So first to work, both parties actually have to have contributed.
Fault essentially uses a breach of duty analysis (E.g. in Gough, it was reasonable
for the girl to rely on the directions of the lorry driver and thus, she didnt
contribute)
Also, with regards to contributed, there is a different concept of causation. You
look at whether Cs act has been relegated to history and if so, theres no
causation (Jones)
o And the court then makes an award that is fair and just
E.g. if one part is more careless than the other, the award is adjusted to reflect
that (Froom v Butcher)
o The problem with contributory negligence is deciding how responsible each party is.
Denning suggests some guidelines in Froom but they arent really followed so really its
up to the judge.
Illegality (4 theories on it)
o It is unfair to allow an illegality defence where C would otherwise be entitled to
compensation at law (Revill)
o Illegality comes in at the stage of deciding whether a DoC exists as something to be
considered under stage 3 of the Caparo test
o There is an illegality defence (Gray v Thames Trains)
o Illegality alters the standard of care C is entitled to expect

Seminar 9: Occupiers Liability

(1) The common law


Standard of care varies with status of entrant
Contractor Invitee Licensee Trespasser

(2) Occupiers Liability Act 1957


(A) WHO IS AN OCCUPIER?
Wheat v Lacon and Co Ltd [1966] AC 552
Facts: The case concerned a brewery which occupied the bottom floor of a pub. The manager and his wife
have the top floor of it where they allow guests to stay. A fatal accident occurs on the stairs. It was
claimed there wasnt a light and the bannister was too short
Judgment: Lord Denning defined occupier as a person who has sufficient control over the premises to the
extent that he ought to foresee that lack of care on his part can cause damage to his lawful visitors.
(B) WHO IS A VISITOR?
Section 1(2) preserves common law position:
for the purpose of the rules so enacted the persons who are to be treated as an occupier and as his
visitors are the same (subject to subsection (4) of this section) as the persons who would at common law
be treated as an occupier and as his invitees or licensees.
(Essentially, invitees and licensees; contractors are included. Importantly, the CL has developed rules
regarding implied licenses e.g. Glasgow corp v Taylor)
(C) COMMON DUTY OF CARE
Section 2(1): An occupier of premises owes the same duty, the common duty of care, to all his visitors,
except in so far as he is free to and does extend, restrict, modify or exclude his duty to any visitor or
visitors by agreement or otherwise.
Section 2(2): The common duty of care is to take such care as in all the circumstances of the case is
reasonable to see that the visitor will be reasonably safe in using the premises for the purposes for which
he is invited or permitted by the occupier to be there
(D) WHAT THE ACT COVERS
Section 1(1):The rules enacted by the two next following sections shall have effect, in place of the rules
of the common law, to regulate the duty which an occupier of premises owes to his visitors in respect of
dangers due to the state of the premises or to things done or omitted to be done on them.
Dangers due to the state of the premises
Fairly self-explanatory (e.g. Wheat v Lacon)
Things done or omitted to be done on the premises?

Fairchild v Glenhaven Funeral Services [2002] 1 WLR 1052 (in the CA): occupancy duty - not a duty
arising from activities being carried out on land
Notes: CA draws a distinction between duties arising from occupancy and duties arising from the
activities that have been carried out. It does it again by looking back at the CL which was developed pre1957 and says that occupiers liability is due to dangers based on the static state of the premises, whereas
activity duties are based on things that have been done on that premises.
With regards to the things done or omitted to be done on them bit, they give the following example: if
somebody has carelessly carried out some repair work on a building and part of the building falls down,
they say that that then would come within the second part of s1(1).
Again Mitchell isnt sure about it because the language of 1(1) seems broader. But the compelling reason
to interpret it narrowly is because if we interpret it broadly, even situations where for instance an invitee
is attacked by the dog of the owner may be covered by the act. This could make occupiers liability have
the potential of being a huge exception to the general principle of a lack of liability against TPs.
Tomlinson v Congleton BC [2004] 1 AC 46
Facts: In May 1995: C dived into the water of an artificial lake at a county park and hit his head on the
sandy bottom, leaving him tetraplegic. He subsequently brought proceedings against Congleton Borough
Council under the Occupiers' Liability Act 1984 (as a trespasser),
Judgment: Lord Hoffman draws a distinction between a risk that arises out of the state of the premises and
a risk arising out of the act of the claimant. Only the former will have liability under the act.
(E) SPECIAL ISSUES
Section 2(3): The circumstances relevant for the present purpose include the degree of care, and of want
of care, which would ordinarily be looked for in such a visitor, so that (for example) in proper cases
(a) an occupier must be prepared for children to be less careful than adults; and
(b) an occupier may expect that a person, in the exercise of his calling, will appreciate and guard
against any special risks ordinarily incident to it, so far as the occupier leaves him free to do
so.
Section 2(4):In determining whether the occupier of premises has discharged the common duty of care to
a visitor, regard is to be had to all the circumstances, so that (for example)
(a) where damage is caused to a visitor by a danger of which he had been warned by the occupier, the
warning is not to be treated without more as absolving the occupier from liability, unless in all the
circumstances it was enough to enable the visitor to be reasonably safe; and
(b) where damage is caused to a visitor by a danger due to the faulty execution of any work of
construction, maintenance or repair by an independent contractor employed by the occupier, the
occupier is not to be treated without more as answerable for the danger if in all the circumstances
he had acted reasonably in entrusting the work to an independent contractor and had taken such
steps (if any) as he reasonably ought in order to satisfy himself that the contractor was competent
and that the work had been properly done.
(i) Children s 2(3)(a)

Phipps v Rochester Corp [1955] 1 QB 450


Facts: Some children were out on their own playing in a grass field which was part of Ds building site.
There was a ditch which had been dug into which C fell. There were no warnings regarding the potential
danger. Though it was an obvious danger for an older child or an adult, this was not the case for C. Also,
it was reasonable for children to be playing in that open space.
Judgment: The judge held that the occupier was entitled to assume that either the parents wouldnt let the
children onto the premises at all or that the parents will be there to look after them. So it may be that an
occupier might be able to say that reasonable care of the visitors safety did not require him to take extra
care for kids as he had anticipated/expected/insisted even that the parent should accompany the child onto
the premises.
Notes: Importantly, Devlin J doesnt suggest this will always work. If the occupier is or ought to be aware
that children often play unsupervised in the area, then he does have to take precautions sufficient to allow
the unsupervised child to be safe.
What for instance will that mean about warnings? Little children might not be able to read or understand
the significance of the warnings. This means then that the duties you would owe are different depending
on what you know about the neighbourhood. (E.g. a well-known child truancy neighbourhood with kids
always running about vs one where the kids are always supervised. Does this mean occupiers liability is
altered by the prosperity/social conditions of the area in which the property is located?)
Jolley v Sutton LBC [2000] 1 WLR 1082 (Also important for remoteness of damage)
Facts: Here, the authority is the occupier of a council estate on which there is an old wooden boat which
was in a bad condition that had been abandoned. The Claimants were children who liked the boat and had
a dream they were going to mend the boat and take it to the seaside and sail it. They start to try and do
work to mend the boat. Unfortunately, while doing this work, the boat which is rotten, collapses onto one
of them badly injuring him.
Judgment: Since the council had conceded that it should have removed the boat because of the risk that
children might suffer minor injuries if the rotten planking gave way beneath them, the wider risk of more
serious injury being caused by the condition of the boat, which could have been eliminated without the
council incurring additional expense, also fell within the scope of the council's duty of care. (Highlights
that for remoteness, it doesnt matter if you dont foresee the exact way the damage comes about or the
magnitude. Merely foreseeing the type of damage is enough)
(ii) People there in the exercise of their calling s 2(3)(b)
Essentially refers to people there for a specific purpose. E.g. youd expect an electrician to take care not
to suffer shocks. There are limitations. E.g. if an ambulance man trips over a hold to the floor, is it a risk
incidental to his calling? The courts havent taken a consistent approach. Note that most cases in the area
concern delivery men and ambulance men.
(ii) Warnings - s 2(4)(a): enough to enable the visitor to be reasonably safe
Darby v National Trust [2001] PIQR 372
Facts: C dives into a pond and drowns. He was a competent swimming but the pond was murky, and as
much as 10 feet deep in places. People frequently swam in it but little was done to discourage this only

an inconspicuous sign by the car park. It is suggested that there should be a sign saying that if you dive
into the cold water you might get hypothermia and drown. The question for the court is whether thats an
obvious risk. Its not as trivial as a general sign saying you might drown in water or a sign saying you
may fall off the edge of a cliff for instance. The issue was about the extra risk the cold water carries.
(There is also a further question about a sign warning of the risk of Weils disease)
Judgment: the risks to competent swimmers of bathing in the pond were obvious, and that there was
therefore no duty to warn against swimming. The additional submission by the claimant that the risk of
Weils disease required warning notices, and that had they been present they would have prevented the
claimant's husband from swimming, could not support a claim for damages resulting from a quite
different cause.
(iv) Independent contractors - s 2(4)(b): discharges common duty of care if reasonable to employ
contractor, plus reasonable steps taken to ensure contractor is competent and insured against thirdparty liability
Haseldine v Dawe [1941] 2 KB 343
Facts: Work being done is work to a lift and the independent contractors engaged tell the occupier that the
lift is safe to use even though some parts of it may need replacing. As it is war time, the contractor says
youre unlikely to be able to get these parts anyway so we suggest merely re-greasing these parts.C is
injured when using the lift, the mechanism breaks and the lift plunges to the bottom of the shaft.
Judgment: It is held that D is not liable and the court emphasises that the occupier had done all that was
reasonable. Realise that in most circumstances where s2(4)(b) might apply, occupiers are likely to be in
no position to evaluate whether its been done safely or properly.
An issue that has come up more recently are what steps the occupier has to take to show that it was
reasonable to entrust the work to a particular independent contractor.
Gwilliam v West Herts Hospital NHS Trust [2003] QB 443
Facts: Here, an independent contractor is putting up a splat wall for a charity fair. C is injured when the
splat wall collapses in this fair. She alleges that D should be liable as they did not provide a safe
environment for her as a visitor, and that they failed properly to enquire into the contractor's insurance
status(which had expired just a few days before the event and meant they were not in a position to pay out
the full amount of damages)
Judgment: CA controversially said occupier should take the step of asking the contractor whether they
had insurance.
Notes: Sedley LJ who dissented thought this wasnt enough. He felt that the occupier ought to establish
that the insurance would still be in force at the time any damage might occur.
Payling v Naylor [2004] EWCA Civ 1325 Gwilliam approach to insurance is unusual
Summary: The independent contractor here provided nightclub security on an on-going basis for the
defendant occupier. The general approach to insurance taken here is that here is no requirement to check
on the insurance status at all. Were told that when faced with a one-off transaction, theres a greater onus
to enquire as to the insurance position of the independent contractor but that is reduced when theres an
on-going series of transactions. This does not seem to be a convincing reason to distinguish the cases.

Theres another factor however. Here, the occupier had a good reason to think it was reasonable to entrust
this work to the security firm; contractor was accredited under some sort of police scheme as beinga good
contractor.
Notes: After the 2 cases, as a matter of law, you dont always have an obligation to enquire about
insurance position. And even when you do, detailed investigation isnt required; you just need to ask
whether they have insurance.
(F ) THE COMMON LAW AND TRESPASSERS
CL position applied to those not covered by the 1957 Act. (i.e. no duty owed)Addie v Dunbreck [1929]
AC 1054: comes on property at own risk. This was qualified in 2 ways though.
Doctrine of Allurement
Glasgow Corp v Taylor [1922] 1 AC 44 allurement
The case concerns a bush with berries on it in the botanical gardens. Kids were welcome to go to the
gardens but werent welcome to eat the exhibit. As soon as they did, they become a trespasser for those
purposes. Tragically, the berries were poisonous when consumed. Its said by having this bush with shiny
berries on it, the council was to be liable to him as a visitor even though he was doing something he
wasnt allowed to do. (So the children became implied licensees)
Duty of common humanity
British Railways Board v Herrington [1972] AC 877: duty of common humanity
The HL in Herrington thought that the general rule of no liability for trespassers was too harsh and that
one ought to owe them a lesser duty to visitors. One ought to owe them a duty of common humanity in
relation to obvious high risk dangers. Particularly serious dangers.
Unfortunately in Herrington, 5 speeches are delivered and they take various lines on what this duty
should be. So they didnt succeed in any clear sense of saying what the law was.This caused the LC to
enquire and eventually you get the 1984 Act.

(3) Occupiers Liability Act 1984


(A) THE STATUTORY SCHEME
Section 1(3): An occupier of premises owes a duty to another (not being his visitor) in respect of any
such risk as is referred to in subsection (1) above if:
(a) he is aware of the danger or has reasonable grounds to believe that it exists;
(b) he knows or has reasonable grounds to believe that the other is in the vicinity of the danger
concerned or that he may come into the vicinity of the danger (in either case, whether the other
has lawful authority for being in that vicinity or not); and
(c) the risk is one against which, in all the circumstances of the case, he may reasonably be expected
to offer the other some protection.

Section 1(4): Where, by virtue of this section, an occupier of premises owes a duty to another in respect
of such a risk, the duty is to take such care as is reasonable in all the circumstances of the case to see that
he does not suffer injury on the premises by reason of the danger concerned.
(B) APPLICATION OF THE SCHEME
Donoghue v Folkestone Properties Ltd [2003] QB 1008
Facts: C dived into a harbour in winter and broke his neck. Awarded damages (less 75% for contributory
negligence) at first instances
Judgment: While it was clear that D knew that people dived in the harbour and thus should have done
more, CA held that the phrase the other in s1(3) and another in s1(4) referred to the individual claimant
who suffered injury. Thus, it did not follow that, because the defendant owed a duty to trespassers
swimming in the harbour during the summer of whose presence it was aware, a similar duty was owed to
such trespassers in the winter; that, since on the judge's findings the defendant had had no reason to
believe that the claimant or anyone else would be swimming in the harbour in mid-winter in the middle of
the night, the defendant had owed no duty to the claimant under section 1(3); and that, accordingly,
liability was not established
Tomlinson v Congleton BC [2004] 1 AC 46
Notes: In the case, Lord Hoffman tells us that the circumstances to take into account under s1(3)(C) are
all the factors which you take into account for breach of duty. Namely things like the gravity of the harm
and cost of precaution for instance.
But he also says there are broader issues to take into acct particularly in a case like Tomlinson itself. He
suggests that context influences our sense of whether protection should be offered against the risk.
Hoffman talks about it being an issue of freedom, In Tomlinson itself, the council had already being to
remove the attractive features of the lake to stop people going near it (ploughing up the beach replace it
with rushes etc.)In a sense, protection here interferes with freedom of the general population (to enjoy the
countryside)
Theres another kind of freedom though that Hoffman also talks about namely, the freedom of the
individual to undertake dangerous activities such as diving into a lake or mountaineering. They are
dangerous but we wouldnt think protection should be offered against a risk.
Tomlinson is a landmark case as the judges seem to be fighting against a compensation culture.
SUMMARY
-

Pre 1957, the law on OL was a mess. 4 different categories and the lines were getting blurred etc.
No duty owed to trespassers and different duties for the rest
OL Act 1957 to attempt to codify and harmonise
o Occupier someone who exerts a reasonable degree of control over the premises (wheat
v lacon)
o Visitor s1(2); invitees, licensees, contractors (licenses can be implied Glasgow corp)
o Duty owed? A common duty to take care to ensure reasonable safety. (s2(1), (2) to all
visitors, which can be modified by agreement and to the extent they are allowed in the

premises) in respect of dangers arising from the premises and things done or not
done(s1(1))
Fairchild seems to draw a distinction between activities done on the premises
and the sort of static state of the premises saying only the latter is covered
Tomlinson distinction drawn between risks due to the (static) state of the
premises and risks due to the acts of the claimant
o Exceptions to the common duty
Children (s2(3)(a)) suggests a higher standard might be owed
Phipps if the kids are/ought to be supervised, that is enough to require
only the standard duty (wont always apply and means the duty may vary
by area)
Jolley example of a higher standard being owed. Also useful for RoD.
People exercising their calling (s2(3)(b)). Note the lingering question of the
scope of their calling
Warning signs (s2(4)(a)) enough to make the premises reasonably safe
Darby the area was reasonably safe and though there was an extra risk
of disease which was not adequately warned off, it was not relevant to
the death of the claimant and thus not explored
Independent contractors
Haseldine; reasonable reliance
Gwilliam; reasonable reliance can extend to checking insurance
Payling; seems gwilliam is rare. Requirement may exist with regards to
one-off contracts as opposed to recurring ones but even if it does, it is
only as to enquire whether the contractor has insurance not to take
further steps
Old CL positions remained for trespassers. 2 qualifications though
o Doctrine of allurement (Glasgow corp)
o Duty of common humanity (british railways board)
Law with regards to trespassers changed by the Occupiers liability act 1984
o If D is aware of the danger, knows or has reason to believe C will encounter the danger,
and the danger is one which he might be expected to protect against Then, he has a
duty to take reasonable care to ensure C doesnt suffer the risk
o Donoghue v FolkstoneThe act is limited to the particular claimant and the particular
circumstances
o Tomlinson all the circumstances to be taken into account including freedom of the
individual to do something dangerous and freedom to factor in other factors in deciding
not to put up signs or damage a potentially beautiful place (fight back against a
compensation culture)

Seminar 10: Product Liability


Donoghue v Stevenson establishes that a producer can be liable to the end user. However, it seems to be
felt that negligence liability wasnt enough for product liability. There are some arguments for strict
liability:
-

Manufacturer better placed to bear the risk


Moral enterprise theory

Nonetheless, it seems those reasons didnt convince parliament. The Act comes from the European
directive and their main reasoning was for free movement of goods.

(1) Cause of Action


For liability under the 1987 Act, you need to have: A Product which is defective and it has got to cause
damage.
(A) PRODUCER AND PRODUCT
Consumer Protection Act 1987 s1(2)
Producer, in relation to a product, means
(a) The person who manufactured it;.
(b) In the case of a substance which has not been manufactured but has been won or abstracted, the
person who won or abstracted it;.
(c) In the case of a product which has not been manufactured, won or abstracted but essential
characteristics of which are attributable to an industrial or other process having been carried out
(for example, in relation to agricultural produce), the person who carried out that process;.
product means any goods or electricity and (subject to subsection (3) below) includes a product
which is comprised in another product, whether by virtue of being a component part or raw material
or otherwise; and
(B) DEFECT
Consumer Protection Act 1987 s3
Section 3 meaning of defect
(1) there is a defect in a product if the safety of the product is not such as persons generally are
entitled to expect; and for those purposes safety, in relation to a product, shall include safety
with respect to products comprised in that product and safety in the context of risks of damage to
property, as well as in the context of risks of death or personal injury.
(2) In determining for the purposes of subsection (1) above what persons generally are entitled to
expect in relation to a product all the circumstances shall be taken into account, including(a) the manner in which, and purposes for which, the product has been marketed, its get-up,
the use of any mark in relation to the product and any instructions for, or warnings with
respect to, doing or refraining from doing anything with or in relation to the product;
(b) what might reasonably be expected to be done with or in relation to the product; and
(c) the time when the product was supplied by its producer to another; and nothing in the
section shall require a defect to be inferred from the fact alone that the safety of a product
which is supplied after that time is greater than the safety of the product in question.
(C) LIABILITY FOR DEFECTIVE PRODUCTS
Section 2(2): This subsection applies to

(a) the producer of the product;


(b) any person who, by putting his name on the product or using a trade mark or other distinguishing
mark in relation to the product, has held himself out to be the producer of the product; [e.g.
supermarket brands]
(c) any person who has imported the product into a member State from a place outside the member
States in order, in the course of any business of his, to supply it to another.
Section 2(3): Suppliers who are unable to meet the victims request to identify any other identities in the
supply chain (e.g. wholesalers or manufacturers)
(D) APPLICATION OF THE ACT
A v National Blood Authority [2001] 3 All ER 289
Facts: The claimants contracted hepatitis from blood suffered from the defendants and brought a claim
under the Consumer Protection Act 1987.
Judgment: Held that as the blood was produced and sold (need not be for profit), it is a product. The Act
is not limited to traditional good.
Burton J says that in identifying whether theres a defect, you first identify the harm and then look at
whether the product is a standard or non-standard product.
Also, while he reaffirms that the general idea with the directives were to impose strict liability, he seems
to read down the act. He introduces the idea of relevant circumstances and says that certain factors (e.g.
such as avoidability and social benefit here) are not to be considered as this would cause the test to slip
into one for negligence.
Notes: Despite Burton Js assertion that certain factors were to be disregarded, it seems that these are
factors the courts have taken into account in other cases.
Abouzaid v Mothercare, CA, 20 February 2001 Where C is hit in the eye by the elastic strap. One
factor that influenced the court was that the producer could have done more to prevent the accident. In
other words avoidability.
Bogle v McDonalds [2002] EWHC 490 where C is burnt by hot coffee. One factor against liability is
that it is in the social interest to have coffee served hot.
Non-standard Product
A product which doesnt conform to the manufacturers own standard. Theres almost always going to be
liability unless theres generally something in the expectation of the public that will prevent liability from
arising.
Richardson v LRC Products (2000) 59 BMLR 185 A defective condom leads to an unwanted
pregnancy nonetheless, D is not liable under the CPA.
Standard product

The main problem here is with finding a comparator


General Standards (what people would expect) ?
Palmer v Palmer [2006] EWHC 1284 (QB) at [44]-[54] inadequate instructions render seat-belt device
defective by failing to warn against perils of creating slack in belt
Industry Standards ?
Pollard v Tesco Stores Ltd [2006] EWCA Civ 393 Child resistant cap opened in less than 2 minutes by
an unsupervised toddler. Per LJ Laws . The test as to whether a product had a defect under the 1987 Act
was what persons generally were entitled to expect. Persons were generally entitled to expect that the
bottle in the instant case would be more difficult to open than if it had an ordinary screwtop. Anything
more specific, as a test of public expectation, ran into difficulties. The bottle was more difficult to open
than an ordinary screwtop, though not as difficult as it would have been if the British Standard torque
measure had been complied with. Thus, there was no breach of the 1987 Act
So it seems the obvious comparator would be the same or similar product by a different manufacturer.
This in itself is ridiculed with difficulties. It does seem that you would have to attack the whole design
process. Many factors are considered at this stage with regards to both price and form. So you would have
to consider many aspects such as the look, purpose, and function of the product amongst others. This is
one of the reasons why proving defectiveness of standard products is going to be so difficult.
Products that provide information
No conclusive ruling

(2) Available Defences


Section 4 defences
(1)
(a) that the defect is attributable to compliance with any requirement imposed by or under
any enactment or with any Community obligation; or
(b) that the person proceeded against did not at any time supply the product to another; or
(c) that the following conditions are satisfied, that is to say (i) that the only supply of the
product to another by the person proceeded against was otherwise than in the course of a
business of that persons; (ii) or
(d) that the defect did not exist in the product at the relevant time; or
(e) that the state of scientific and technical knowledge at the relevant time was not such that
a producer of products of the same description as the product in question might be
expected to have discovered the defect if it had existed in his products while they were
under his control; or
(f) that the defect
(i) constituted a defect in a product ("the subsequent product") in which the product
in question had been comprised; and

(ii) was wholly attributable to the design of the subsequent product or to compliance
by the producer of the product in question with instructions given by the
producer of the subsequent product.
(2) In this section, the relevant time means
(a) the time when he supplied the product to another;
Section 6(4):Where any damage is caused partly by a defect in a product and partly by the fault of the
person suffering the damage, the Law Reform (Contributory Negligence) Act 1945 and section 5 of the
Fatal Accidents Act 1976 (contributory negligence) shall have effect as if the defect were the fault of
every person liable by virtue of this Part for the damage caused by the defect.
S(4)(1)(e)
This was tried in A v National Blood (Art 7(e) was tried rather. Check)
-

It failed because they did know that hepatitis existed in some batches.
What they couldnt do was eliminate it from all as they didnt really have a way to determine
which batches had hepatitis without screening all the blood which was expensive and timeconsuming.

It was tried in Mothercare as well


-

It was accepted that the risk was not heard of not easily foreseeably however any limitations were
not due to the state of scientific and technical knowledge; it was held that accident reports did
not constitute technical knowledge and D could have discovered the defect through practical
testing.

European Commission v United Kingdom [1997] 3 CMLR 932 - state of the art defence
NOT a question of what is commonly done re safety in industry (or eg practicability of safety)
knowledge of expert in the sector
scientific knowledge is not linear most advanced knowledge.
accessibilityexample of information published only in Manchuria in Chinese

Seminar 11: Private Nuisance


(1) Statutory nuisance (L&O 681 but we will lecture you on this)
Environmental Protection Act 1990, part III

(2) Public nuisance


A-G v PYA Quarries Ltd [1957] 2 QB 169
Tate & Lyle Industries Ltd v GLC [1983] 2 AC 509
Corby Group Litigation Claimants v Corby BC [2009] QB 335

(3) Private nuisance


Private nuisance is the unreasonable interference with the claimants enjoyment of his or her land.
(A) A TORT AGAINST LAND
As is clear from the definition of nuisance, it is a tort against land.
St Helens Smelting Co v Tipping (1865) 11 HLC 642
Hunter v Canary Wharf Ltd [1997] AC 655

Dust generated in the construction of the Canary Wharf tower has caused in the inhabitants to
develop respiratory problems, promoted asthma in children living in the area
Claims brought against construction of Canary Wharf tower by landowners and their spouses and
children, people who have been lodgers in the places affected by the dust

Why cant they sue?

Actions in private nuisance are usually concerned with interference with the enjoyment of land.
If we allow people who do not have an interest in land to claim, it turns into a personal tort
Court tries to keep it distinct from the tort of negligence
Relies on an article by Newark who traces the history of tort of nuisance (Assize of Nuisance).
He concludes that nuisance should be confined to interference with enjoyment of land
Court adopts this traditional approach

Lord Cookes dissent

Lord Cooke took the approach that people who lived on land, especially as a home, people should
be entitled to sue, particularly children
Under the UN Treaty on Children, children were entitled to a safe environment to grow up
Highlights that the HL in the majority was taking a very traditional kind of approach

The fact that liability for nuisance is confined to people who have the right to exclusive possession means
that we also have to think a little carefully about the damage that you can sue for. What kinds of damage
is this owner or tenant entitled to sue for?

E.g. let say one of the owner of tenants suffered asthma from exposure to the dust, what would
their claim have formed under the tort of private nuisance?
The claim is for the damage to the value of the land, people are less inclined to buy the land or
lease it because of the dust in the air
Loss in amenity value of the land cannot use the land in a manner that you expect to deal with it
Claim is not for the personal injury suffered the claim for personal injury is in negligence (Lord
Hoffmann)

Notes: In Hunter, it was affirmed that both material and physical damage and loss of amenity amount to
interference with property interests.
Only people with proprietary interest in land entitled to sue for private nuisance and only for interests
relating to land property damage or loss in amenity value, not personal injury (Hunter v Canary Wharf).
How did the HL define the class of proprietary interest in land?

Those who are entitled to exclusive possession of land


Typically, the owner and the occupier as tenant
But not the licensee, who has permission to be on the land but has no legal interest in land does
not have standing to sue (cf Khorasandjian v Bush licensee granted standing)

BUT NOTE
It is possible that with the implementation of the Human Rights Act 1998 and article 8 of the ECHR,
individuals without a proprietary interest in their home could bring a claim in nuisance. This is hinted in
McKenna v British Aluminium Limited [2002] (note that it was merely a striking out application).
Also, note definition home for the purposes of article 8 in Khatun v United Kingdom [1998] is
relatively wide. (includes for example children, tenants)
(B) REASONABLE USER
It is clear we are concerned with an unreasonable interference. There are certain common sense factors
we consider:
-

Extent of the interference


Duration of the interference
The timing, particularly with regards to noise

Beyond that, there are certain specific categories we need to think about
(i) Locality/coming to the nuisance
-

Locality is only relevant in the case of amenity damage (St Helens Smelting Co)
Also, what might be a nuisance in one locality might not be so in the other (Sturges)
Note that the nature of a locality may change with planning permissions. (Watson v Croft-Promo
Sport Ltd)
It is no defence to say that something was always done (Miller, Sturges) However, the fact that
the claimant came to the nuisance may be considered, particularly at the remedy stage.

St Helens Smelting Co v Tipping (1865) 11 HLC 642


Facts: D began extensive smelting works on its property which was situated within a mile and of C and C
alleged that the fumes from the work had caused damage to trees and shrubs on Cs land.
Judgment: Jury found for C. Upheld on appeal to HL.
Notes: This case sets out an important distinction between activities which cause material injury to
property, in which case locality is irrelevant, and those which case sensible and personal discomfort, in
which case locality is relevant.
Sturges v Bridgman (1879) 11 Ch D 852
Facts: A confectioner had for more than twenty years used a pestle and mortar in his back premises, which
abutted on the garden of a physician, and the noise and vibration were not felt as a nuisance and were not
complained of. But in 1873 the physician erected a consulting-room at the end of his garden, and then the
noise and vibration became a nuisance to him.
Judgment: It was held to be a nuisance and it was no defence to say that he had always done it.
Notes: Importantly, here Thesiger LJ says what would be a nuisance in Belgrave Square would not
necessarily be so on Bermondsey i.e. residential area vs industrial area. (For a good modern example see
Baxter v Camden LBC (p648)).
Miller v Jackson [1977] QB 966
Facts: Developers build houses very close to the boundary of a cricket ground and as a result, balls
occasionally fly into Cs garden.
Judgment: There is liability in nuisance however, C only gets damages, not an injunction.
(ii) Sensitive claimants
-

The classic thinking was that the claimants sensitivity isnt something that can increase the
liability of D (Robinson)
More recently, it has been suggested that liability should since on whether the interference was
reasonably foreseeably (Network Rail Infrastructure v Morris)

Robinson v Kilvert (1889) 41 Ch D 88


Facts: C stored brown paper which is particularly sensitive to heat. Heat rises from Ds premises
underneath.
Judgment: CA said theres no liability because the fact that Cs property was a bit warm wouldn't
normally be an interference, and it certainly wouldn't be an unreasonable one.
Network Rail Infrastructure Ltd v Morris [2004] Env LR 41

Facts: C owns a studio close to the track and has some music recording devices. The recording process is
interfered with by some trackside electronic equipment owned by D. So it does seem like C is a sensitive
claimant.
Judgment: CA decides to take a different approach. They say you look instead at whether the interference
was reasonably foreseeable and here it wasnt.
Notes: Some commentators have criticised this test as it would mean that so long as a sensitive claimant
tells D about the interference, therell be liability in nuisance. It seems unlikely that the CA would have
intended this.
(iii) Malice
-

If a defendant acts with malice, that might turn an otherwise reasonable act into a nuisance
(Christie v Davey)

Christie v Davey [1893] 1 Ch 316


Facts: Concerns neighbours. C is a music teacher. D sends a letter complaining about the music but no
action was taken. D then bangs on the wall and makes mocking noises whenever the music occurs so C
brings a claim in nuisance.
Judgment: The motive for which the interference was taking place was made relevant to the question of
whether the interference was reasonable or not.
Hollywood Silver Fox Farm Ltd v Emmett [1936] 2 KB 468
Facts: Rifle shots are fired as close to the fox pens as possible with the aim being to cause the foxes to
give birth to a stillborn baby or to eat their own young.
Judgment: While firing a shotgun in the countryside is normally acceptable, the motive made the act a
nuisance.
Contrast the above 2 cases with Bradford Corp v Pickles
Bradford Corp v Pickles [1895] AC 587
Facts:Here, C has a reservoir on the land and water flows into it through Ds land. D wants C to buy his
land but C isnt interested. As such, D alters the flow from the water on the spring from his land so its no
longer feeding the reservoir on Cs land. He effectively cuts off the water supply.
Judgment:HL said this doesnt give rise to liability in nuisance even though he was doing it quite
deliberately to case this nuisance to the ds land. The reason they give is that regardless of the intention,
the act is lawful and thus permissible.
Notes: On the face of it, it is hard to reconcile Bradford with the other decisions. Mitchell says the thing
to focus on is probably that there are special rules dealing with water rights. The HL in pickles draw on a
range of authority even going back to Roman law about peoples rights to use water in any self-interested

way particularly as far as the law of nuisance is concerned. As a neighbour, you have no right to insist that
your neighbour permit water to flow into your land.
Olliphant feels the best explanation is suggested in Winfied; namely that the right to make noise on ones
land is relative, whilst the right to alter percolating water is absolute. Where a relative right is in issue it is
appropriate to consider malice. The problem arises, in the absence of clear judicial authority, in defining
what relative rights are. (Note for example that the right to make noise and smells are relative)
(iv) Public benefit, planning permission, and statutory authority
Public Benefit
The fact that Ds activity is for public utility doesnt prevent it also amounting toa nuisance (Bamford).
However, if an activity is for the public benefit, that may be a consideration taken into account in
determining the reasonableness of the defendants use of his land. Further, if an activity is in the public
interest, a different remedy may be required as an injunction may not be appropriate. (Dennis)
Bamford v Turnley (1862) 3 B & S 66
Facts: D was burning bricks on his land resulting in smoke and smell that affected C. D won but one of
the arguments was that D should not be subject to a claim in private nuisance as his activity was for the
public benefit.
Notes: Bramwell B for his part felt that an activity was only for the public benefit if there was a net
benefit after paying compensation and that the argument did not hold water.
-

Although the authorities suggest that public benefit is not an absolute defence, it seems that it
may be a consideration taken into account in determining the reasonableness of the defendants
use of land.
If private rights are to be extinguished in favour of the general public this has generally been
thought to be a matter for parliament

Dennis v Ministry of Defence [2003] 2 EGLR 121


Facts: C, the owner of a country estate adjoining an RAF base, claimed that the noise amounted to a
nuisance. D argued that the public benefit prevented it from being characterised as a nuisance.
Judgment: Buckley J held that whilst public benefit could not be used to justify conduct that would
otherwise be a nuisance, it may be relevant to determining the remedy that should be granted. (If nuisance
was found, an injunction would have been the appropriate remedy because the conditions for the award of
damages in lieu of an injunction, set out in Shelfer, would not have been met. Nonetheless, an injunction
clearly would not have been appropriate. Thus, Buckley J effectively extended the circumstances, well
beyond Shelfer, when damages could be granted in lieu of an injunction.)

Statutory authority

Defendants have immunity in respect of proceedings for nuisance in respect of inevitable consequences
from carrying out statutorily permitted activity. (Note that the immunity only extends to inevitable
interferences) Alternatively, no action lies for doing that which the legislature has authorised, unless done
negligently.
Allen v Gulf Oil Refining Ltd [1981] AC 1001
Facts:D built and operated an oil refinery under the authority of the Gulf Oil Refining Act 1965. C
claimed she was adversely affected by noxious odours, vibrations, and offensive noise levels emanating
from the refinery, as well as alleging that she and members of her family lived in fear of explosion at the
refinery. D pleaded statutory authority.
Judgment:HL held that the statute expressly or by necessary implication gave authority to construct and
operate the refinery, and that this authority conferred on the defendant immunity from proceedings for
any nuisance which might be the inevitable result of constructing a refinery on the land, however
carefully sited, constructed, and operated.
Notes:The question with statutory authority is thus one of statutory construction.

Planning permission
Planning permission can change the character of the area and if so, you assess the reasonableness of the
interference with respect to the new character. (Gillingham)
Also, where the planning permission doesnt change the character of the area, the fact that permission has
been granted in itself does not prevent liability in nuisance. (Wheeler)
Gillingham DC v Medway (Chatham) Dock Co Ltd [1993] Ch 19
This was a case involving the granting of planning permission for D to operate a 24h commercial wharf in
an area that had formerly been part of the Chatham Royal Dockyard, Buckley LJ hinted that planning
consents relating to major redevelopments that of necessity prejudiced some local inhabitants might act as
a defence to subsequent private nuisance actions by those adversely affected. InHunter, it has been made
clear that the comments in Gilingham mean no more than suggesting that the character of a
neighbourhood can change in line with planning permissions.
Wheeler v JJ Saunders [1996] Ch 19
Planning permission in the case concerned the intensification of pig farming, Held that it did not change
the character of the neighbourhood
(C) FORESEEABILITY AND FAULT
One of the confusing issues in nuisance is whether it is a fault-based tort. The short answer to this
question is that it depends, first, on what remedy is sought, and, secondly, on what is meant by fault. Most
authors agree that where C is seeking an injunction, the question of fault is irrelevant. This is because at
the time the case is brought, the defendant knows that his conduct is causing the claimant inconvenience,
and the only question for the court is whether the inconvenience amounts to a nuisance for the purpose of
preventing such conduct in the future.

(i) Where D creates the nuisance


Cambridge Water Co Ltd v Eastern Counties Leather plc [1994] 2 AC 264
D spilt some chemical on its floor which over time, got into the water supply of C, forcing them to move
to a new water source and stuff. (facts not so relevant here) Judgment for D though.
Here, the principle above is mentioned (where C is seeking an injunction, the question of fault is
irrelevant). However, it is also made clear that a failure to take reasonable care in carrying out an activity
on land is not necessary to establish liability in nuisance.
Nonetheless, a defendant will only be liable for reasonably foreseeable consequences of the unreasonable
user. This might be construed as a form of fault-based liability.
Note that an alternative view is that there is really no difference between the fault requirement in
negligence and that in nuisance, at least where damages are sought rather than an injunction.
Delaware Mansions Ltd v Westminster CC [2002] 1 AC 321
(case concerned liability for encroaching tree roots)
Also, case describes damage caused by a natural occurrence as nuisance but, at least where damage to
property is claimed, it is in reality negligence.
This is one of the cases where it has been recognised that the occupier of a land may owe a duty to act
positively to prevent harm to his neighbour. Lord Cooke says at 333 The level nuisance or negligence is
treated as of no real significance. In this field, I think the concern of the common law lies on working out
the fair and just content and incidents of a neighbours duty rather than affixing a label and inferring the
extent of the duty from it
Network Rail Infrastructure Ltd v Morris [2004] Env LR 41
EM interference case. C failed as the interference was not reasonably foreseeable as a result of the
defendants conduct and as such, couldnt amount to an unreasonable use of land.
(ii) Third parties / natural events
Sedleigh-Denfield v OCallaghan [1940] AC 880
Facts:a TP (who was trespassing) constructed a pipe on Ds land to channel water from a ditch for
irrigation. Though it was meant to have a grill to prevent debris entering, this was built negligently. The
pipe got blocked and flooded (twice), and damaged Cs land.
Judgment:Though D didnt know about it, as a member of his order did, he was held to know.
Notes:Essentially established that a nuisance was continued or adopted if the defendant knew or ought to
have known of the existence, or the possibility of a nuisance on his land and failed to take reasonable
steps to bring the nuisance to an end.
It is now well established that an occupier can be liable for a nuisance on his land created by the acts of a
third-party.

It has also been used to impose liability on occupiers for the actions of their predecessors in title;
and to support the liability of a landowner in nuisance where a TP uses the defendants land as a base
from which to engage in activities off the defendants land which interfere with Cs use and enjoyment of
his land. (note that though liability above would seem to arise under the principle in sedleigh, it seems to
arise only in nuisance in respect of amenity damage (such damage not being actionable in negligence).
A similar fault-based liability is imposed where the alleged nuisance is constituted by an isolated escape.
(see more on 661)
Goldman v Hargrave [1967] 1 AC 645
Case where a lightning strike set a tree on Ds property on fire. D took action to put it out but this was
inadequate and the fire spread to Cs property and caused extensive damage.
Applying the principle in sedleigh, PC held that the occupier of land owed a duty to his neighbour to
remove any hazards, whether natural or man-made. The duty was based on the occupiers knowledge of
the hazard and foresight of damage to his neighbour if he failed to act. The standard of care required of
the occupier was what it was reasonable to expect of him in his circumstances, and since the duty to act
was imposed on the occupier, the individual resources of the occupier were to be considered in assessing
whether his response was reasonable. The PC felt that liability, if any, arose in negligence.
Leakey v National Trust [1980] QB 485
(Barrow Mump case on 662) Again reiterates that the test of deciding whether what the defendant did
was sufficient or reasonable is somewhat subjective in that the resources available to D can be taken into
account.
NB. The result of these cases is that where the alleged nuisance is caused by a natural occurrence an
action may be commenced in either nuisance or negligence, but that liability will be determined according
to principles applicable to the law of negligence.
The only exception is where the damage C alleges is not sufficient to found an action in negligence but
would be in nuisance.

(4) The rule in Rylands v Fletcher (1866) LR 1 Exch 265


The rule imposes strict liability for the escape of dangerous things on land. For liability, four
requirements need to be satisfied.
1. The object in question must be a dangerous thing. I.e. something likely to cause damage if it
escapes.
2. The dangerous thing mustve been accumulated on the defendants land. (Rylands). This is
sometimes phrased as being a non-natural use of the land (Read, Transco)
3. dangerous thing must escape from Ds land
4. dangerous thing must cause damage to the claimant
Note:No liability for escapes caused by acts of God or malicious third parties; no liability for death or
personal injury

Rylands v Fletcher [1868] (HL)


Facts: D engaged independent contractors to construct a reservoir to supply water to his mill. This was
built over abandoned mine shafts which collapsed due to the weight of the water, causing water to flood
into Cs colliery. D had not be negligent and there was no basis for a claim in private nuisance as D had
taken reasonable care to select a competent and experienced independent contractor.
Judgment: D was held liable on the basis that the person who for his own purposes brings on his land and
collects and keeps there anything likely to do mischief if it escapes, must keep it at his peril, and, if he
does not do so, is prima facie answerable for all the damage which is the natural consequences of its
escape. Lord Cairns described the required use of land as non-natural use.
Non-natural use of land
This particular aspect has presented the most difficulty. The debate exists as it is unclear whether this
refers to a use of the land which isnt attributable to natural processes or rather, whether by non-natural
use we should understand it as some extraordinary or unusual use. (Per Read and Transco, it seems the
latter view is preferred)
Read v J Lyons & Co [1947] AC 156
Facts:Concerned an explosion in an armament factory. Question was whether the explosion could give
rise to Rylands liability.
Judgment: HL said that the storage of explosives in an armament factory is a natural use of land.
Notes: It was acknowledged that what may be regarded as dangerous or non-natural will vary according
to the circumstances taking into account the circumstances of the time and the practice of mankind
Transco plc v Stockport MBC [2004] 2 AC 1
Facts: Escape of water from a pipe owned by the defendant LA caused an embankment to collapse, which
exposed a gas pipe, thus necessitating expensive emergency remedial work by the claimant.
Judgment:HL did not accept that this fell within Rylands on the basis that the supply of water through
pipes was normal and routine and not something that presented a particular hazard. The risk presented by
any particular activity had to be considered by contemporary standards. As the pipe carried no more risk
of fracture leading to the escape of water than any other pipe, it could not be considered a non-natural use
of land. Lord Hoffman noted that damage to property caused by leaking water was a risk against which
insurance was available, which supported the conclusion that this situation did not meet the high
threshold of exceptional risk arising from non-natural use that is required if a claim under Rylands is to
succeed.
Liability
It is clear that the tort is one of strict liability. However, in Cambridge Water, Lord Goff made it clear that
liability here was only strict in the sense that the defendant would be liable even if he had taken steps to

prevent the escape occurring and that the type of damage suffered by the claimant must still be reasonably
foreseeable.
Cambridge Water Co Ltd v Eastern Counties Leather plc [1994] 2 AC 264
Facts: the defendant company was a leather manufacturer which used chemical solvents in the tanning
process. These chemicals were stored in drums on the defendants premises. Following new European
regulations, tests were carried out on the claimants water and it was found to be polluted by the
chemicals from the tannery as spillages had leaked into the water. At first instance, the claim based on
Rylands was dismissed on the basis that there was not a non-natural use of land due to the amount of time
that the tannery had been in operation and the industrial area in which it was located. The CA rejected this
argument and held that the storage of chemicals was a non-natural use of land and found the defendant
liable for the damage caused on a strict liability basis. This issue was reconsidered by the HL.
Judgment: Lord Goffexamined the precise wording inRylands as evidence that the principle required at
least foreseeability of the risk as a prerequisite to recovery of damages. He went on to say that the tort
was strict liability only in the sense that D would be liable for the consequences of escape even if he had
taken steps to prevent it occurring. Lord Goff made reference to Wagon Mound (No 1) and concluded that
Rylands required foreseeability by D of the relevant type of damage.

Seminar 12: Intentional interference with


the person
(1) Cause of action
(A) M EANING OF INTENTION
HL was invited to import a Woolin style meaning of intent but declined to do so saying the aims of the
different strands of law are different. But if anything, one would think intent might have a wider
definition where mere compensation is concerned rather than punishment. It is also important that the
action be intended as if not, it is merely negligence.
(B) ASSAULT
Definition: An unlawful act which causes another person to apprehend the infliction of immediate,
unlawful force on his person.
Stephens v Myers (1830) 4 C& P 349: (D walking towards C at some church meeting waving a clenched
fist and was stopped while still 2-3 seconds away from being able to conduct the battery). D was liable for
assault so the test for assault requires the reasonable apprehension of an immediate an unlawful touching.
It need not be an action possibility.
R v Ireland [1998] AC 147: (Repeated silent phone calls) HL hold that silence can constitute an assault.
They note that it is contextual as well. E.g. if you had two people in a room and one said nothing, that
would not be sufficient.

(C) BATTERY
Definition: The intentional and direct (unlawful) application of force to a person.
Hostility?
Required in Wilson v Pringle [1987] QB 237but disapproved in Re F [1990] 2 AC 1; So it is not
required. This is in line with the criminal law definition. (See R v Brown)
A positive act
Innes v Wylie (1844) 1 Car & Kir 257: D (a policeman) was standing passively blocking a doorway.
Held that it was not a battery and that a positive act was required.
Directness Requirement
This requirement seems to be absent in the criminal law. With regards to trap cases and the like, it is
unclear whether they satisfy the directness requirement for the tort. The case law seems to indicate that
the danger must be created in a relatively short time period before damage for it to be actionable.
(D) I NTENTIONAL INFLICTION OF PHYSICAL OR EMOTIONAL HARM
Wilkinson v Downton [1897] 2 QB 57
Facts: D decides to play a practical joke on C and falsely tell her that her husband was seriously injured in
an accident. This caused her to vomit, her hair to turn white, as well as suffer other serious and permanent
psychiatric and physical harm.
Judgment: It was held that a person who has wilfully done an act calculated to cause physical harm to the
claimant and has in fact thereby caused physical harm to her has provided a good cause of action.
Notes: So here, you need to specifically intend harm as opposed to intending the unlawful application of
force and for liability, damage has to be suffered (unlike assault and battery which are actionable per se)
Khorasandjian v Bush [1993] QB 727
Facts: Stalking case where D is constantly harassing C amongst other things by calling her at home. The
harassment is so intense shes at risk of developing a psychiatric injury in the future.
Judgment: The CA use Wilkinson to grant an injunction to prevent the tort happening.
Notes: After Khorasandjian, it seemed like Wilkinson might grow into a useful tort but the need for it to
grow has been reduced by the Protection from Harassment Act 1997.
Protection from Harassment Act 1997 ss1, 3, 7
S1(1) essentially prohibits a course of conduct which amounts to harassment or he ought to know
amounts to harassment.

In s1(2) For the purposes of this section, the person whose course of conduct is in question ought to
know that it amounts to harassment of another if a reasonable person in possession of the same
information would think the course of conduct amounted to harassment of the other.
Importantly, per s7(3), a course of conduct must involve conduct on at least two occasions.
If you are guilty of harassment, s3 allows for civil remedies such as damages and an injunction. Its also a
criminal offence.
Majrowski v Guys and St Thomas NHS Trust [2007] 1 AC 224
Facts: An employee claims to be harassed by his departmental manager in the NHS in breach of s1 of the
Harassment Act and sues the employer claiming theyre vicariously liable.
Judgment: Per Lord Nicholls of Birkenhead. Where the quality of the conduct said to constitute
harassment is being examined, courts will have in mind that irritations, annoyances, even a measure of
upset, arise at times in everybody's day-to-day dealings with other people. Courts are well able to
recognise the boundary between conduct which is unattractive, even unreasonable, and conduct which is
oppressive and unacceptable. To cross the boundary from the regrettable to the unacceptable the gravity
of the misconduct must be of an order which would sustain criminal liability under section 2.
Notes: HL confirms that the tort imposes vicarious liability.
Ferguson v British Gas Trading Ltd [2010] 1 WLR 785
Facts: C had changed to another gas provider. For a period of about five months after the transfer, D was
attempting to get some payments from C which she did not owe due to a computer error. D made a
striking out application.
Judgment: Although harassment under section 1 of the 1997 Act was both a crime and a tort that did not
modify in any way the constituents of the civil wrong; that, since the individual was expected to tolerate a
certain amount of annoyance, the impugned course of conduct had to be grave, in that it was oppressive
and unacceptable, before either the criminal or civil law would intervene; and that the defendant's conduct
as alleged by the claimant was capable of satisfying that test and so the claim should go to trial.
Notes: The key point here is that the act is drafted broadly enough to take account of this sort of
harassment. So it covers all sorts of harassing conduct and is not limited to stalkers or threats of violence.
Wainwright v Home Office in the CA: [2002] QB 1334, [1]-[15], [41]-[51] (Lord Woolf CJ), [74]-[86]
(Buxton LJ); in the HL: [2004] 2 AC 406
Facts: C and his mother are strip-searched when visiting his brother in prison. C was so severely affected
by his experience as to suffer post-traumatic stress disorder. Mrs Wainwright had suffered emotional
distress but no recognised psychiatric illness.
Mental Element Question
Per the CA, it seems that you required subjective recklessness. D needs to be aware that hes created this
risk of harm. It will not suffice that D was objectively reckless.
This is not fully uncontested as while Lord Woolf took this view, LJ Buxton wasnt so sure and started off
by thinking the test was one of objective recklessness. Nonetheless, the test does seem to be subjective.

Role of Wilkinson today


Lord Hoffman stresses that the scope of Wilkinson has been reduced today. It is important that the case
was decided in 1897 as there was no liability under negligence for psychiatric injury at the time. Over the
course of the 20th century however, liability for PI does come to be recognised.
Lord Hoffman personally feels that Wilkinson shouldve been swept into the law of negligence and has
only been kept alive by barristers and textbook writers. However, while he isnt convinced the tort has a
role in itself independent of negligence, he does not rule out the potential for a tort based on or amending
Wilkinson which would cover ground not currently protected by the law of negligence or the Protection of
Harassment Act.
This would have to be a non-recognised psychiatric injury such as grief (which would not be covered by
negligence) and would have to be the result of a one-off incident. (So as to not be covered by the Act). He
does go on to say that if you had such a tort, youd have to be very careful about how intention was
defined and thought that the current subjective recklessness test of intention in Wilkinson would not be
good enough. Theres also the point of how people regularly suffer distress and humiliation at work.

(2) Defences
(A) CONSENT
Requirements:
1. C must have the capacity to consent
Re MB [1997] 2 FLR 426 The claimant must be capable of understanding and retaining the information
and using it in making a decision. (most relevant in medical cases)
2. Consent must be real (C must understand the broad nature of the touching)
Chatterton v Gerson [1981] QB 432 (misleading information)
Facts: C consents to a nerve treatment operation. Unfortunately, more nerves than intended were killed off
which resulted in her having numbness in parts of her body. The issue was whether her consent wasnt
real or whether it had been destroyed by not being properly informed about the risk.
Judgment: The judge analysed it in 3 steps. He came to the conclusion that the failure to inform her didnt
prevent the consent from being real.
He said generally, fraud vitiates consent; e.g. fraudulently induced sexual intercourse would amount to
rape. However he said, for a situation like this involving medical treatment, the lack of information to
amount to preventing the consent being real has to go on beyond mere carelessness.
He concluded that what youre looking for is whether C understood the broad nature of the act. (Here, she
understood it was a medical procedure therefore, she consented to the touching.)
Mitchell thinks the conclusion is right and the test he came up with is right but he doesnt think the judge
can be right when he talks about at the start of his judgment that fraudulently induced consent to
intercourse amounts to rape. Thats not what the criminal law says.
R v Dica [2004] QB 1257

D charged in relationship to various acts of sexual intercourse hed had with women when he knew he
was HIV positive. Its made very clear by the CA that theres no question of him being guilty of rape. The
Fact he represented that in some way he wasnt suffering from HIV didnt take away the consent to the
intercourse.
So the key test is really whether you understand the broad nature of the touching. Its not so much a
question about being given misinformation. Only misinformation that means you no longer understand
the broad nature of the touching can count as taking away consent.
3. The consent given must cover the application of force
Covered in the case of check
Boy needed to be admitted to a hospital to have his tonsils out and owing to misadministration, instead he
was circumcised. The consent to having your tonsils out doesnt extend to anything the doctors may do on
you while youre under GA.
Note that consent can be implicit/implied.
Blake v Galloway [2004] 1 WLR 2844 (boys throwing sticks)
[21] In a sport which inevitably involves the risk of some physical contact, the participants are taken
impliedly to consent to those contacts which can reasonably be expected to occur in the course of the
game, and to assume the risk of injury from such contacts. Thus, for example, in the context of a fight
with fists, ordinarily neither party has a cause of action for any injury suffered during the fight. But they
do not assume the risk of a savage blow out of all proportion to the occasion. The man who strikes a
blow of such severity is liable in damages unless he can prove accident or self-defence
(B) SELF -DEFENCE
A person may use such force as is reasonable to protect against an actual or perceived threat of harm
against themselves or another person. The force used must be proportionate to the threat.
Ashley v Chief Constable of Sussex Police [2008] UKHL 25, [2008] 1 A.C. 962 HL held that in cases
where the assailant had acted in a mistaken belief that he was being attacked, this mistaken belief must be
honestly and reasonably held in order for self-defence to be available against a civil claim in assault or
battery.
(C) CAPACITY/NECESSITY
Mental Capacity Act 2005 ss 1-6, 24-27 (note this is an important statute and it is essential that you are
familiar with these sections)
Section 1: principles
s. 1(2) A person must be assumed to have capacity unless it is established that he lacks capacity.
s. 1(3) A person is not to be treated as unable to make a decision unless all practicable steps to help him to
do so have been taken without success.

s. 1(4) A person is not to be treated as unable to make a decision merely because he makes an unwise
decision.
Section 2: people who lack capacity for the purposes of this Act:
s. 2(1) a person lacks capacity in relation to a matter if at the material time he is unable to make a decision
for himself in relation to the matter because of an impairment of, or a disturbance in the functioning of,
the mind or brain
Section 3: inability to make decisions.
s. 3(1) a person is unable to make a decision for himself if he is unable
(a) to understand the information relevant to the decision,
(b) to retain that information,
(c) to use or weigh that information as part of the process of making the decision, or
(d) to communicate his decision (whether by talking, using sign language or any other means).
Section 4: assessment of best interests
s. 4(2) The person making the determination must consider all the relevant circumstances
s.4(3) He must consider
(a) whether it is likely that the person will at some time have capacity to in relation to the matter in
question,
s. 4(4) He must, so far as reasonably practicable, permit and encourage the person to participate,
s.4(6) He must consider, so far as is reasonably ascertainable
(a) the persons past and present wishes and feelings (and, in particular, any relevant written statement
made by him when he had capacity),
(b) the beliefs and values that would be likely to influence his decision if he had capacity

Section 5: Acts in connection with care or treatment


Person doing such an act is treated as if claimant had consented with capacity (ie, not liable) if:
before doing the act, D takes reasonable steps to establish whether P lacks capacity in relation to the
matter in question, and
when doing the act, D reasonably believes
that P lacks capacity in relation to the matter, and

that it will be in Ps interests for the act to be done.

Section 24. Advance decisions to refuse treatment.


Definition: decision must be made after person has reached age 18, and has capacity

s.24(3) P may withdraw or alter an advance decision at any time when he has capacity to do so.
s.24(4) A withdrawal (including a partial withdrawal) need not be in writing.
s.24(5) An alteration of an advance decision need not be in writing

Section 25. Validity and applicability


s.25(1) An advance decision does not affect the liability which a person may incur for carrying out or
continuing a treatment in relation to P unless the decision is at the material time(a) valid, and
(b) applicable to the treatment.

s.25(2) An advance decision is not valid if P(a) has withdrawn the decision at a time when he had capacity to do so,

(c) has done anything else clearly inconsistent with the advance decision remaining his fixed decision.

s.25(3) An advance decision is not applicable to the treatment in question if at the material time P has
capacity to give or refuse consent to it.

s.25(4) An advance decision is not applicable to the treatment in question if(a) that treatment is not the treatment specified in the advance decision,
(b) any circumstances specified in the advance decision are absent, or
(c) there are reasonable grounds for believing that circumstances exist which P did not anticipate at the
time of the advance decision and which would have affected his decision had he anticipated them.

s.26 Effect of advance decisions


s.26(1) If P has made an advance decision which is
(a) valid, and
(b) applicable to a treatment,
the decision has effect as if he had made it, and had had capacity to make it, at the time when the question
arises whether the treatment should be carried out or continued.
s.26(2) A person does not incur liability for carrying out or continuing the treatment unless, at the time, he
is satisfied that an advance decision exists which is valid and applicable to the treatment.

(D) G ENERAL PRINCIPLES OF NECESSITY


F v West Berkshire Health Authority [1990] 2 AC 1 (L&O 89-94). These principles, although they predate
the Mental Capacity Act 2005, are relevant for two reasons. First, the Act only applies to certain types of
incapacity (s.2). Second, Lord Goffs discussion indicates how the courts might interpret the Act.
Here, Lord Goff gave examples of necessity that went beyond medical treatment;
For example, if I push you out of the way of an oncoming vehicle you werent aware of, Id have a
defence of necessity if you sued for battery.
The point about the defence of necessity is that it arises in situations where its impractical to
communicate with C. So whereas ordinarily, if you were going to touch someone, youd obtain their
consent to do so, in the lorry pushing example, theyd be run over if you did so.
So the defence of necessity can apply more broadly than medical care or treatment
Some specific points
-

If D doesnt believe act is in Cs best interests but is doing it, he wont have a defence under s5.
If D is wrong about it and it is in Cs best interest to have the treatment done, thatd be covered
by the common law principles of necessity as those arebsaed on whether its actually in Cs best
interest and now whether D believes it is.
So where D wrongly believes its not in the best interests to have the treatment done, hed have
no defence under the Act but you should have a defence under the common law if it actually is in
Cs best interests.

Re MB [1997] 2 FLR 426 (L&O 96-99) (helpful to show the way the courts may interpret the 2005 Act).
(E) CONTRIBUTORY NEGLIGENCE?
Law Reform (Contributory Negligence) Act 1945 ss 1 and 4 see Seminar 8 handout for text
Murphy v Culhane [1977] QB 94
Standard Chartered Bank v Pakistan National Shipping Corporation [2003] 1 AC 959

Bici v Minstry of Defence [2004] EWHC 786 (QB)


Q about whether it could apply in situations where D has committed an intentional tort (standard
chartered case concerned intentional tort of deceit)
Standard Chartered Bank v Pakistan National Shipping Corporation [2003] 1 AC 959
check
There, HL concluded that the defence of contributory negligence would only be available where itd
previously been available prior to 1945. They said the 1945 Act was designed to change only the effect of
a defence in contributory negligence, not to increase the number of situations where the defence is
available.
Prior to 1945, contributory negligence was a complete defence and hence its unsurprising that prior to
1945, courts didnt accept it as a defence to intentional torts.
However, after 1945, though we have the Standard Chartered decision thats probably the most
authoritative, we have 2 other cases that seem to suggest something else.
Murphy v Culhane [1977] QB 94and Bici v Ministry of Defence [2004] EWHC 786 (QB)
Murphy v Culhane [1977] QB 94
C is essentially killed as a result of Cs actions. There are suggestions that contributory negligence could
apply for intentional torts. However, its merely an interlocutory appeal and a CA decision which the HL
in Standard Chartered refuse to say was correctly decided.
Bici v Ministry of Defence [2004] EWHC 786 (QB)
Soldiers taking part in United Nations peacekeeping operations in Kosovo owed a duty to prevent
personal injury to the public and had breached that duty by deliberately firing on a vehicle full of people
when they had no justification in law for doing so
Elias J found that regardless, the claimants could not be said to have contributed to the damage.
Nonetheless in dicta, he considered contributory negligence in tort and suggests that it should be allowed.

Ultimately inBici, the Standard Chartered case isnt discussed and the point is dealt with in brief. Murphy
is a CA decision which the HL in standard chartered refuse to say was correctly decided.
So though the case law is patchy, the leading case is Standard Chartered. And unless the SC says the case
is wrong, the position seems to be that contributory negligence isnt available for intentional torts such as
battery.
When do these defences apply?
They clearly apply for battery and assault.
With regards to Wilkinson, consent is clearly applicable however, there is no case law for the other
defences so its hard to say.

Seminar 13: Defamation


(1) The cause of action
Cause of action consists of the publication of defamatory words; Publication here means making known
this imputation to one or more people other than the claimant.
(A) DEFAMATORY MEANING
The CA in Berkoff said there are several separate tests for deciding whether the words have a defamatory
meaning
-

Causes C to be shunned
Exposes C to hatred, ridicule, or view with contempt
Lowers the claimants reputation in the estimation of right-thinking people (dominant test)

Right-thinking people?
Byrne v Dean [1937] 1 KB 818
Facts: There, the implication was that C had reported to the police the presence in a golf club of some
illegal gambling machines. C sued for defamation. Because C as a member of the golf club was now
being shunned or being treated less favourably by members at the golf club.
Judgment: Its held theres no liability because right thinking people wouldn't think less of a person who
reported a criminal offence to the police.
Notes:
Dissent; its true right thinking people might not think less favourably on someone for reporting illegal
activity; but nonetheless, right-thinking people might shun disloyalty. The allegation here may be read not
as imputing reporting to the police but as imputing disloyalty
-

Shows importance of right thinking people test


Focuses on the question of how defamatory language should be interpreted. Whats the
appropriate approach to take? In particular, what do we do with allegations that can be read as
making a range of different defamatory accusations against our claimant?

Berkoff vBurchill [1996] 4 All ER 1008


Facts: A film critic reviewing the movie Frankenstein said that C was hideously ugly and that
Frankenstein was marginally better looking. (Second time shed made a similar comment).
Theres this general idea that simply to say something crude or abusive shouldnt amount to defamation as
its not the kind of thing thatll lower their reputation in the eyes of right thinking people.But with the
breadth of the test extending to ridicule, its possible situations like this might be covered.

His typical roles were minor characters or a gangster/villain. Typically characters which werent expected
to be pleasant looking. In that sense, it probably wouldn't really affect his job.
Judgment:CA suggested that people might be put off from seeing his work because they thought it was an
unpleasant experience, that might be enough to count as defamation. (so the finding in this case is simply
that it can be put to the jury.)
(B) I NTERPRETATION
Words can have both defamatory and non-defamatory meanings. How does the court interpret them?
Lewis vDaily Telegraph [1964] AC 234
Facts: D publishes an article saying that Cs company is being investigated by the fraud squad. C sues on
the basis that the words were understood by readers as implying that hed been guilty of fraud.
Judgment: HL said were concerned with the ordinary meaning of words (so were dealing with the
ordinary reasonable reader) and thus, guilt of fraud could not be interpreted from the article. (And so there
was no case to go to a jury)
Notes: HL said it was quite important that we shouldnt read the words in an ultra-technical or lawyer
like way. But even if that were the case, people shouldnt be allowed to sue because those reading the
article had read too much into it.
What if readers have special knowledge allowing them to give the statement a different meaning to that of
the ordinary reasonable reader?
Cassidy v Daily Mirror[1929] 2 KB 331
Facts:Here, D published a photograph of a man and a woman with the caption, Mr Corrigan with miss x
whose engagement has been announced; the people in the photo saw the photographer at a horse race
and asked them to take this picture telling him theyd just became engaged to be married.
However, Mr Corrigan was actually married and was Mr Cassidy. His wife, C, brought the claim on the
basis that her friends saw the picture in the mirror and when they saw the caption, that lowered Mrs
Cassidys reputation because the friends thought she was the one not married to the man.
Judgment:CA held that this defamatory meaning was actionable. In doing so, there were 2 things to note
about their finding
Notes:
-

The daily mirror didnt know about Mrs Cassidy so it couldnt have known that people would
have read this statement as being defamatory. In defamation, that doesnt matter though as
liability is strict. So no fault liability.
It was only possible because the claimant was permitted to set out the story behind the photo.
How it was that people came to give these words a defamatory meaning which they would not at
first sight bear.

That technique as setting out a special meaning not on the surface of the words, is the
technique of pleading a true innuendo. Thats where you explain why words that dont
on their surface explain what they mean actually have that meaning. You explain special
circumstances. Special knowledge possessed by a class of readers.

A false innuendo by contrast is where C in his formal pleading explains what the words mean but that
explanation is simply amplifying or expanding what the obvious meaning is.
In Byrne, the problem is the class of people were not right thinking people but in Cassidy, the problem
was that the people were right thinking people who had some particular special knowledge who had a
reason which was invisible to ordinary people.
So in Cassidy the class is if you like right thinking, specially informed people.
(C) LIBEL AND SLANDER
(Libel is in permanent form whereas slander is impermanent, typically speech) Libel is actionable per se
while slander requires proof of special damage with 4 exceptions (indictable criminal offence, serious
contagious disease, professional incompetence, allegations of unchastity)
Why the distinction?
Thorley v Lord Kerry (1812) 4 Taunt 355
The court recognises that it is not possible to justify the libel/slander distinction on the basis that written
words necessarily cause more harm or reach a larger group. Also, as defamation is now a strict liability
offence, the idea of written words having more malice is irrelevant. The court here says though that ultimately
there is a lot of historical support for the distinction and as such, we have to keep it.
2 Exceptions Though
Theatres Act 1968 s 4(1)
Words spoken in the course of a theatrical performance are to be treated as libel (though interestingly, not
those spoken during rehearsal)
Broadcasting Act 1990 s 166(1)
Words spoken in a broadcast (essentially any programme) are to be treated as libel.
(D) WHO CAN SUE FOR DEFAMATION?
Any individual named or referred to in the ordinary meaning of the words (Lewis) can sue.
Companies:
Jameel v Wall Street Journal Europe sprl [2007] 1 AC 359
Facts:Discussed below. However, one of the arguments raised was that for a company to sue in defamation, it
should have to prove actual damage.
Notes: The HL by a majority reject that argument. So a company can sue in the same way as an individual.

Hoffman dissent: companies arent like people where a company has no immortal part no soul or sort of
human dignity and as a result, it ought not to have remedies for defamation in the same way an
individual.
Groups:
KnupffervLondon Express Newspaper Ltd [1944] AC 116
Facts: In an article, D accused a group (the young Russians party) of trying to help the Germans in their war
effort against Britain. C was the leader of the party and sued on the basis that people would think of him
when they saw a reference to the party.
Judgment: HL said that the question was whether the reference to a group in itself implied a reference to
every member of the group. One of the things youd consider was the size of the group. They mentioned a
Scottish case with 7-8 people mentioned where all could sue and contrasted it with a large group for which it
was clear all couldn't. Here, he was not allowed to sue.
Notes: There are some peculiarities with Knupffers case in particular which may have denied him a claim.
Government:
Derbyshire County Council vTimes Newspapers Ltd [1993] AC 534
Facts: The Sunday times had published articles alleging that the council had mismanaged its pension funds.
The council itself along with the council leader and the finance leader sue for defamation.
Judgment: The HL holds that no liability is possible in defamation to governmental bodies (like a LA or a
governmental department). So a governmental body like a LA, a government department, cannot sue for
defamation. (Though here, the 2 individuals can)
Notes:The main reason for this has to do freedom of speech; the importance of being able to debate, criticise,
evaluate, the activities of elected authorities. Its important to notice though that HL was careful to say that
that didnt mean you could say what youd like about government and never be liable. Could be liable either
under malicious falsehood or if sued by individuals concerned.
(E) STRICT LIABILITY
E Hulton & Co vJones [1910] AC 20
Facts: The newspaper produced what was intended to be a fictitious sketch about Mr Artemis Jones, an
upright member of the community at home but a drunken lout on holiday. An actual Artemis Hones, a
barrister in Peckham, sues for defamation.
Judgment: The claim succeeds. The HL says that the intention of the publication is not important but rather,
what is understood by it.
NewsteadvLondon Express Newspapers [1940] 1 KB 377
Facts: D publishes an article reporting that Mr Newstead from Camberwell who was convicted for bigamy.
This was actually true however, the claim is brought my another Mr Newstead (same first name as well) also
from Camberwell.
Judgment: The claim succeeds. So wordsintended true of their target may also be libellous.

Notes: As a result of these 2 decisions (amongst others), it became media practice to institute a process called
negative checking; its where you go through various listings, directories, and any other sort of information
you can find to illuminate a coincidence of names. Particularly important for fiction
OShea v MGN [2001] EMLR 943
Facts: D published a pornographic advertisement featuring an AV model who bore a striking resemblance to
C and indeed, those who knew C thought shed gone into porn. C brings a claim for defamation.
Judgment: The claim fails; it is not possible to do anything like negative checking with pictures and imposing
strict liability in such circumstances would be inconsistent with art. 10. Secondly, there is no pressing social
need for the protection of look-alikes.

(2) Defences
(A) I NNOCENT DISSEMINATION
Defamation Act 1996 s 1 Essentially, where D is neither author, editor, nor publisher of the defamatory
material, where he neither knew nor ought to know that the material was defamatory, hed have the defence
available. (Distributors etc extends to electronic publications)
Notes:
Note the wording. It is important to realise that the defence can be lost even where:
-

D is informed that the material is defamatory


D suspects that the material might be defamatory.

(B) JUSTIFICATION
D only needs to prove that the claim is true in substance. This leads to two particular points
(i)
(ii)

Repetition rule (Stern)


Ensure that the actual thrust of the defamatory claim, confined to its scope, is proved.
(allegations of general bad conduct are not proved by a single instance of unrelated bad
behaviour and vice-versa)

SternvPiper [1996] 3 All ER 385


D merely reports something another person has said. Mere repetition without actual proof does not suffice for
justification.
Bookbinder v Tebbit [1989] 1 All ER 1169
Facts:Claims were made by D (a prominent conservative leader) that C (leader of a labour CC) had
squandered public money by overprinting its stationery. C adduces many other instances of public money
being wasted during his term in office.
Judgment: CA holds that it is not possible to justify the allegation by pointing to other alleged instances of
squandering public funds
Scott v Sampson (1882) 8 QBD 491 General evidence of a bad reputation is not justification.

Defamation Act 1952 s 5 Essentially reverses the old CL position


In an action for libel or slander in respect of words containing two or more distinct charges against the
plaintiff, a defence of justification shall not fail by reason only that the truth of every charge is not proved if
the words not proved to be true do not materially injure the plaintiffs reputation having regard to the truth of
the remaining charges
(C) FAIR COMMENT
4 elements
-

Comment (as opposed to fact)


Fair
In the public interest
Cant be malicious (reintroduces an element of fault)

Comment
Clearly covers subjective opinions. But how far does this idea of a comment go?
Telnikoff vMatesuvitch [1992] 2 AC 343
Facts:Concerns a letter published in a newspaper in response to a previous letter. C sues alleging that the D
made it seem that he was advocating a racialist recruitment exercise when in fact he wasnt. The question
then was whether the statement was a statement of comment or a statement of fact. If you put the letters side
by side, it seemed like a comment.
Judgment: HL said you had to look at the publication as it would be seen by the ordinary reasonable reader
and therefore, it had to be looked at in isolation.
Can something count as a comment if presented as an inference/deduction from factual evidence?
Historically yes, though note Singh and Spiller
British Chiropractic Association v Singh [2010] EWCA Civ 350
Facts: D wrote an article in which through deduction, he presented certain statements about the
(in)effectiveness of chiropractic treatment. At first instance, it was held that to qualify as fair comment, the
statement had to be a statement of opinion, not a statement of fact, even if the statement of fact is presented as
a conclusion from other facts.
Judgment: The CA failed to specifically deal with this point but allowed the defence of fair comment.
Notes:
With regards to malice, it is clear Singh holds these opinions but it is possible the newspaper editor might not
have the defence available. This was not explored.
One thing the CA focused on here was the context of the debate. CA seemed to be saying that in this kind of
(epidemiological)area, the debate is necessarily about matters of opinion. So even though CA says its very
much based on the facts, it suggests courts are going to be open to recognise as comments, things which
arent expressly flagged up as being comments. So it seems that while its got to be a statement of opinion,
you dont need to preface it expressly as long as the nature of the subject matter shows its your opinion.

Spiller v Joseph (2010) (UKSC)


Here, again the court doesnt specifically deal with this question. However, Lord Phillips says he is doubtful
that presenting something as a deduction from true facts is enough to make it qualify as a comment. (In the
case, the defence of fair comment was allowed)
Fair
There seem to be 2 strands to this
1. Sufficient foundation of fact
a. Unclear exactly how much foundation. London Artists indicates that it must be enough for a
person to reach the same conclusion as the actual defendant. It is emphasised that fair
comment should allow for people to be prejudiced.
2. Factual foundation must be identified
a. Unclear as to the degree of identification.It used to be thought that you had to set out the
factual foundation in sufficient detail to allow the reader to decide whether they agreed with
the opinion.
b. SC in Spiller v Joseph said that the test now is simply that you might identify at some
general level the factual basis of the comment.
i. Allows people to comment on public events or occasions. One of the examples
given is people commenting on the performance of contestants of a reality TV show
the next day. So if something is publicly (well-)known, youre not required to set
out the factual basis for your comment in detail.
Matter of public interest
Well-expressed in London Artists. Public interest is to be understood broadly and even extends to
developments in show business.
London ArtistsvLittler [1969] 2 QB 375
Facts:D wrote and published a letter suggesting the claimants had taken part in a plot to force the end of the
run of a successful play with D was producing. D alleged that C1 had written on behalf of all four claimants
of the case identical notices to leave the cast. The defendants pleaded fair comment and qualified privilege.
Judgment: The defence of fair comment failed (The fate of plays publicly performed and the comings and
goings of the actors are matters of public interest on which fair comment can legitimately be made. However,
the facts on which that comment is based must be truly stated, and an allegation that there is a plot between
the principal actors to end the run of a play is not a comment but a statement of fact which must be justified.)
Qualified privilege failed as the publication was not privileged.
Notes:
Lord Denning: Three points on the defence of fair comment
-

Was the comment made on a matter of public interest?


In order to be fair, the commentator must get the facts right
Were there any facts on which a fair minded man might honestly make such a comment?

Comment must not be malicious

Malice in this context merely means that D does not hold the opinion he expresses. (It had a different
meaning in the rest of defamation) See Singh
Defamation Act 1952 s 6 In an action for libel or slander in respect of words consisting partly of
allegations of fact and partly of expression of opinion, a defence of fair comment shall not fail by reason only
that the truth of every allegation of fact is not proved if the expression of opinion is fair comment having
regard to such of the facts alleged or referred to in the words complained of as are proved
(D) Q UALIFIED PRIVILEGE
(i) Common Law
Toogood v Spyring (1834) 1 CM & R 181 For a statement to be privileged it must serve the "common
convenience and welfare of society" and be "fairly warranted by any reasonable occasion or exigency",
meaning that there must be a duty to publish to the particular audience which, in turn, must have a
corresponding interest in receiving the material.
Notes: The general idea behind qualified privilege is that the person disclosing it has some sort of legal or
moral duty to publish it. More recently, there is this idea of the Reynolds defence which isnt strictly qualified
privilege but more generally, a defence that the information published is in the public interest.
Reynolds v Times Newspapers Ltd [2001] 2 AC 127
Facts: D had published an article in Ireland stating that Reynolds, the former Irish Prime Minister, had misled
the Irish Parliament. This article was subsequently published in the UK but did not include the explanation
that Reynolds had given for the events, which had been printed in the original article. Reynolds brought an
action for defamation. The defences of justification and fair comment were unavailable, given the factual
nature of the article. The question for the HL was whether the defence of qualified privilege should be
extended to cover the mass media.
Judgment and Notes: In his judgment, Lord Nicholls provided a list of 10 criteria against which attempts to
use the Reynolds defence should be judged:
The elasticity of the CL principle enables interference with freedom of speech to be confined to what is
necessary in the circumstances of the case. This elasticity enables the court to give appropriate weight, in
todays conditions, to the importance of freedom of expression by the media on all matters of public concern.
Depending on the circumstances, the matters to be taken into account include the following. The comments
are illustrative only.
1. The seriousness of the allegation. The more serious the charge, the more the public is misinformed
and the individual harmed if the allegation is not true.
2. The nature of the information, and the extent to which the subject-matter is a matter of public
concern.
3. The source of the information. Some informants have no direct knowledge of the events. Some
have their own axes to grind, or are being paid for their stories.
4. The steps taken to verify the information.
5. The status of the information. The allegation may have already been the subject of an investigation
which commands respect.
6. The urgency of the matter. News is often a perishable commodity.
7. Whether comment was sought from the claimant. He may have information others do not possess
or have not disclosed. An approach to the plaintiff will not always be necessary.
8. Whether the article contained the gist of the claimants side of the story.

9. The tone of the article. A newspaper can raise queries or call for an investigation. It need not adopt
allegations as statements of fact. (HL indicates that a sensationalist tone would count against D)
10. All the circumstances of the publication taken as a whole, including the timing.
The list is not exhaustive. The weight to be given to these and any other relevant factors will vary from
case to case. Any disputes of primary fact will be a matter for the jury, if there is one. The decision on
whether, having regard to the admitted or proved facts, the publication was subject to qualified privilege
is a matter for the judge. This is the established practice and seems sound. A balancing operation is better
carried out by a judge in a reasoned judgment than by a jury. Over time, a valuable corpus of case law
will be built up.
In this particular case, the HL felt the standards hadnt been met. Crucially, the source of the allegation
was one of Cs political rivals that bore him an obvious grudge. Secondly, the article in the English
edition did not contain Cs version of events. Thus, it did not satisfy the requirements of responsible
journalism. By contract, the Irish version did. It explained Reynolds side and wasnt completely reliant
on this biased source of information.
Jameel v Wall Street Journal Europe SPRL [2007] 1 AC 359
Facts: This concerned a publication in the European edition of the Wall Street Journal. The WSJ reported that
the Saudis were cooperating with the US in the investigation of terrorism and had thus frozen the accounts of
various Saudi companies, naming some in the article.
Judgment:
Lords Bingham/Hope: reciprocity between duty and interest was essential
Lords Hoffmann/Hale: Reynolds was not a privilege but a new public interest defence
Hale: it is not useful to analyse the case in terms of a duty/interest; it is in truth a defence of publication in the
public interest
Hoffmann: the categories of responsible journalism are not tests to pass
Per curiam: You dont have to show that the specific defamatory allegation is a matter of public interest. Its
enough if you show that he subject matter of the story is a matter of public interest.
Further developments since Jameel
A key development is this idea of reportage; it gives a defence to someone who has simply reported what
someone else has said in a neutral tone, provided the defendant has not adopted the allegation being reported.
It is important to realise that the repetition rule still applies however, you can make a clear, balanced, and
neutral report of allegations made in the public interest and provided the allegation is not adopted, the duty to
verify the underlying truth is less pressing.
Malice
-

If malice is shown on the part of the defendant, the privilege is lost


Malice for qualified privilege means that your predominant motive was not to perform the duty
underlying the privilege.
This is likely to have little relevance to the Reynolds privilege

The Reynolds privilege simply addresses the single question of whether a responsible journalist
would see publications as justified, true, or false

Horrocks v Lowe [1975] AC 135


Per Lord Diplock:

QP is lost on occasions where it is misused malice is one of these occasions


The motive with which the defendant on a privileged occasions made a statement is crucial
The speech at the meeting was upon matters of local concern, and the evidence of malice was a
lack of apology
The lack of apology is a tenuous evidence of malice, for it is consistent with a continuing belief in
the truth of what one has said
The crucial finding is the belief in truth thus there can be QP Defence operating here

(ii) Statute/Reports
Defamation Act 1996 s15
s15 is structured in a particularly unusual way. It gives a defence of qualified privilege to 2 distinct categories
of publications. Essentially, its concerned with reports of public proceedings/meetings/events. But it splits
those public events into 2 categories. One category gains the defence of qualified privilege simply by falling
w/in the list of requirements
e.g. reports on shareholders meetings count as being protected by qualified privilege.
2nd cat only counts as being protected by qualified privilege provided C has been given the opportunity to
reply.
e.g. reports of proceedings in overseas legislatures are protected by qualified privilege but only provided you
offer C the opportunity to comment/reply and you publish that comment with the report.
One particular issue
McCartan Turkington Breen (a firm) v Times Newspapers Ltd [2001] 2 AC 277
S15 gives qualified privilege to reports of public meetings. While a press conference isnt a public meeting in
the obvious sense of the word, the fundamental purpose is to publicise things being said.
HL said that a press conference was an important vehicle for promoting the discussion and furtherance of
matters of public concern and thus can be afforded QP under statute.
(E) ABSOLUTE PRIVILEGE
This arises only in narrowly defined circumstances and applies irrespective of Ds mental state. There are 2
broad categories in which it works
1. Statements made in court (whether by judges, witnesses, representative, or parties)
2. Bill of Rights 1689, art 9 which provides that free speech in parliament should not be questioned
outside of it.
Defamation Act 1996 s 13

This gives an option to wave parliamentary privilege. It essentially exists to allow MPs to sue.
There is one other exception outside these two;
Chatterton v Secretary of State for India in Council [1895] 2 QB 189
This seems to cover communications made by one high level official to another. (Here, it was between the
secretary of state for the colonies to the undersecretary; i.e. a minister to his deputy) Here, the courts said the
defence of absolute privilege should apply. However, this has been rejected in subsequent cases.
Ultimately, its a powerful defence available in narrow circumstances.

(3) Remedies
(A) COMPENSATORY DAMAGES
Courts and Legal Services Act 1990 s 8
This was introduced to regulate damages as it was felt juries were awarding them too generously. Under s8,
CA has the power to substitute its own award for a jurys award. It was hoped that by doing this and by
emphasising the importance of the right to freedom of speech, that the general level of the awards would be
brought down.
JohnvMGN [1996] 2 All ER 35
Claim by sir Elton against the mirror in which the jury had awarded very high damages. CA said the award
was too high and should be reduced and in the future, jurors should be told to think about the comparable
awards for personal injury damages.
For instance, LordBingham giving the leading judgment, talked about the kinds of awards that would be
made for non-pecuniary loss where you had for instance, loss of a limb or permanent disability. And that
these kinds of awards should not be lower than the awards made for defamation
So in assessing compensatory damages, juries and indeed judges now have totake into account not only what
they think the extent of the loss might be but also if you like, consistency across the law.
Consider awards for serious and even less serious injuries and try to even out the awards.
(B) EXEMPLARY DAMAGES
Can damages be punitive? One of the situations where exemplary damages can be awarded is where theres
some proof that D calculated that it was profitable to commit the tort.
(C) M ITIGATION
What can a defendant do to keep the award level down?
Scott v Sampson (1882) 8 QBD 491
Whilst D is allowed to show C already had a bad reputation, D is not allowed to prove that C has done things
that means he deserves to have a bad rep. So if you sue for defamation, and youve got some nasty secrets in
your past that people dont know about, D isnt entitled to bring that up.

Thats an important restriction not only in itself, but also, because it has a profound effect on theway the
defence of justification operates. Recall, we discussed thered be a conflict b/w parties as to how the words
would be interpreted.
The rule in Scott means that theres an incentive for D to give words the widest meaning possible. So its
almost strange that its often D arguing for a wide meaning as they have evidence of some wrongdoing by a C
so they want a wide meaning. They cant use it under mitigation because of Scott so they can only bring it in
under the defence of justification. So theyll say the words have a broad meaning which includes the kind of
stuff we have information on.
(D) O FFER OF AMENDS
Defamation Act 1996, ss 2-4
A new statutory mechanism. Essentially, its s2-3 that focuses on the question of an offer of amends as a
remedy. s4 provides that if youve been offered amends (d has offered tomake amends in some way, and
youve rejected it) that can be a defence for D provided D neither knew nor ought to have known that the
matter was defamatory.
(E) I NJUNCTIONS
Dealt with in seminar 16 on remedies in general

14. Privacy
(1) No right to privacy in English law
Malone v Metropolitan Police Commissioner [1979] Ch 344 The police wire tapped the defendant
without authority from any statute and convicted him of an offence on the basis of evidence obtained as a
result of the wiretap. It was held that he had no right to privacy to be protected.
Though there was no general right to privacy, there was a patchy but somewhat effective coverage
through torts such as trespass, nuisance, defamation and the harassment (after the act and Wilkinson
before it). There wasnt complete coverage though and the notable gaps are highlighted in the two cases
below.
Kaye v Robertson [1991] Fleet Street Reports 62
Facts: C was an actor who had a bad accident and suffered brain damage or some other serious injury.
Reporters from a tabloid newspaper get into his room despite sign saying no visitors etc. However, he lets
them in so theres no remedy in trespass. They then interview him and they publish the interview and
pictures as being an exclusive in the Sunday sport.
Judgment: in the CA, C wins using malicious falsehood and it was the fact that D said theyd obtained his
permission for the exclusive that was the falsehood. It caused him damage as it would put off other
tabloids from potentially bidding for his story.

Notes:We can see the basis for the decision doesnt go to the heart of the problem as really, he was injured
and wanted to protect the fact that hed been disturbed while resting in this bad state.
Other torts were tried and failed; defamation, trespass to the person (flash), tort of passing off (requires C
to show theres been some confusion in the market created by Ds act)
Importantly, the CA remarked that it seemed to be a shame that we dont have a tort to do with privacy
that could give C the right kind of remedy.
A rather different attitude towards a tort of privacy is taken in Wainwright.
Wainwright v Home Office [2004] 2 AC 406 (Lord Hoffmann)
Facts: Concerned the claimants being stripped searched when visiting someone in prison. Besides
claiming under Wilkinson, they sought to claim a tort of invasion of privacy committed against them by
virtue of the strip-search.
Judgment: Hoffman rejects this idea that we need a general tort of privacy. Hes unconvinced by the US
comparative materials that have been produced. For him, it seems the importance of protecting privacy is
something the law of tort should bear in mind as it develops its own autonomous categories of tortious
liability. Hes against creating or recognising a new tort.
Notes: However, in other strands of privacy, broadly, dealing with unwanted publishing, we have seen a
large expansion of protection of privacy through the expansion of the tort of breach of confidence.
The main reason the courts have expanded the tort of breach of confidence to provide a remedy for breach
of privacy is to do with the influence of the convention rights. The courts are clear to say that they arent
compelled to act in this manner due to the HRA but rather, are simply taking account of convention case
law.

(2) Breach of confidence protecting privacy before the HRA 1988


Note: 3 elements to the classic tort of breach of confidence. (i) element of confidence, (ii) obligation of
confidence, (iii) unauthorised use of the confidential information (element, duty, breach)
Woodward v Hutchins [1977] 1 WLR 760 (Denning)
Facts: The claim here is in respect of a book written by a man who used to be the tour manager of a
collective of pop musicians including Tom Jones (Woodward. Jones is his stage name) one incident
concerns tom jones being drunk and unpleasant on a commercial airliner.
Judgment:CA says theres no tort committed in respect of that info because its not confidential. All the
people on the flight had the chance to see jones behaving in this way.
Notes:So it seems that for information to have this element of confidence, it has to be secret or at least
known only to a very small number of people.

Something private on the other hand doesnt necessarily need to be known to a small number of people.
So this idea of confidentiality seems to mean something narrower or at least different to what privacy
means.
Stephens v Avery [1988] 1 Ch 449
Facts: C and D were close friends who freely discussed matters of a personal nature on the basis that it
was secret and disclosed in confidence. D passed on to a newspaper, details of the plaintiff's sexual
conduct. C brought an action claiming damages on the grounds that the information was confidential and
was knowingly published in breach of the Ds duty of confidence. The defendants applied to strike out the
claim as disclosing no reasonable cause of action but the master refused. The defendants appealed.
Judgment: Held, dismissing their appeal, that although the courts would not enforce a duty of confidence
relating to matters which had a grossly immoral tendency, information relating to sexual conduct could be
the subject of a legally enforceable duty of confidence if it would be unconscionable for a person who had
received information on the express basis that it was confidential subsequently to reveal that information
to another. The defendants had not shown that there was no legal basis for the plaintiff's claim.
Notes: So while the typical example of an obligation of confidence is in the commercial context,
particularly with regards to trade secrets and the like, it is clear it can potentially arise in other
circumstances.

(3) The transformation of the traditional tort of breach of confidence


(A) THE TRANSFORMATION IN CAMPBELL V MGN
Campbell v MGN [2004] 2 AC 457 (Lords Hoffmann and Hope, Lady Hale)
Facts: Naomi Campbell is spotted and photographed, coming out of a narcotics anonymous meeting, by
the paparazzi. The photos are published along with a story and the complaint is about the disclosure in
the mirror.
Judgment:
Rather helpfully, the elements of that story are broken down into 4 separate bits by HL.
(i) Alleges the fact of the addiction
(ii) Alleges shes being treated for drug addiction
(iii) Shows that shes attending NA
(iv) Information is given about the treatment methods used at NA
In relation to the first 2 aspects of that story, the fact of the treatment and the fact of the addiction,
Campbell accepts they were entitled to publish it as its exposing her previous lie about not being a drug
addict.
The Issue then becomes whether there should be liability in relationship to the other 2 elements
highlighted namely, the details of treatment and a photograph of her leaving NA.

The majority hold there is liability (3-2 split); minority being Lords Hoffman and Nicholls.
Even though its a split decision, all the members of the HL agree on the basic principles to be used in
deciding cases like this. Oliphant is probably right to say Nichollss speech is the most helpful.
Nicholls explains the tort of breach of confidence has expanded to the extent that its not appropriate to
talk about a tort of breach of confidence in a sense. He says its more accurate to think of it as the misuse
of private information. He says there are 2 stages to assessing whether this tort of misuse of private
information has occurred.
(i) Whether there is a reasonable expectation of privacy. If so, you then go on to consider whether
there are factors that outweigh that expectation of privacy (stage 2)
(ii) Proportionality (typically balancing expectation of privacy with freedom of expression)
Lord Nicholls says you cant presume that one prevails over the other. These rights are of presumptively
equal weight. So what youve got to look at is the extent of the interference with the privacy and the
nature of the speech being set up against it.
Nicholls point here is that because it was justifiable to publish the fact of addiction and the fact that shes
receiving treatment, the rest just follows from it.
Note:
With regards to the reasonable expectation of privacy, this was not really elaborated. Lord Nicholls
suggested the test was whether the publication would be highly offensive to C.
-

Threshold requirement for the tort would depend on sensitivity of C


You run together the 2 questions Nicholls had been careful to sort out
It might mean that someones expectation reduced over time

Lady Hale suggests focusing on whether the person publishing the information knew or ought to have
known it was private
-

The same information could be legitimately published by one person but not by another.

Later cases have made it clear it is an objective test about the reasonable expectation of privacy which the
law might protect.
(B) EXPECTATION OF PRIVACY
The test for the (reasonable) expectation of privacy
Author of a Blog v Times Newspapers [2009] EMLR 22
The court declined to restrain a newspaper from revealing the identity of a blogger, which it had deduced
from publicly available sources, because that information did not have about it the necessary quality of
confidence, nor did it qualify as information in respect of which the blogger had a reasonable expectation
of privacy, essentially because blogging was a public activity.

The court adopted a two-stage approach when addressing claims based upon the publication of allegedly
private information in contravention of the European Convention on Human Rights 1950 art.8, asking
whether the claimant had a reasonable expectation of privacy in relation to the particular information in
question and, if so, whether there was some countervailing public interest such as to justify overriding
that prima facie right. The test for privacy was an objective one.

The problem with a subjective test is further highlighted by;


Murray v Sunday Express [2008] EWCA Civ 446
Facts:C here is J.K Rowlings child aged 19th months and the claim is based on a photo taken with a long
ranged lens of him being pushed in his pram.Its a striking out application.
Judgment: The decision from the CA is that it is arguable that the right to privacy under art8 is engaged.
Notes:
We may question the type of expectations 2 year olds have about privacy. This in turn may lead us to
question Lord Nicholls formulation as he surely couldn't have intended to get witness statements from 2
year olds. Indeed, in some ways wed think the childs privacy is particularly worth of protection as
theyve had such little choice as to whether to expect this attention.
So it seems to be an objective test and in a sense more objective than other tests in that it shouldnt take
Cs age into account in deciding what the reasonable expectation is. So this idea of the reasonable
expectation is a theme that comes through in the cases but we have to interpret intelligently. Its a
normative, objective test. Not one that turns on asking C what theyd actually expect and if answer is
nothing they lose protection.
(C) THERE ARE CASES WHERE CERTAIN COMPETING INTERESTS OUTWEIGH THE EXPECTATION OF
PRIVACY

Note: It is clear that public interest can outweigh the expectation of privacy (Woodward, Campbell). But
with this test were focusing on balancing rights and the typical countervailing right is freedom of speech
(Article 10)
S12 of the HRA seems to tilt the favour a little bit towards freedom of speech and freedom of press.
However, that doesnt mean the burden is always on the claimant.
HL in Campbell said you dont treat all types of speech in the same way. Lady Hale suggests a sort of
hierarchy. Political speech is at the top of the list and is most likely to outweigh the right to freedom of
privacy. While idle celebrity gossip will be significantly further down.
As we get further down the list, the courts seem to be looking at the degree of interference with the right
to privacy as compared with the rights to freedom of speech. With regards to things less than political
speech;

Murray v Sunday Express (CA)


It is worth noting that the courts have taken a strong view when it comes to protecting the rights of
children. Besides the fact that were not talking about an individuals subjective state of mind when we
consider a reasonable expectation of privacy, it seems that it will be relevant at the proportionality stage
with regards to children. In Murray, the CA seems particularly concerned to protect private information
about children. So it seems that publishing private information about children is less likely to satisfy that
test of outweighing the article 8 rights.
Mosley v News Group Newspapers Ltd [2008] EWHC 1777 (QB)
Facts:Mosley was exposed as having a Nazi orgy with some prostitutes. The Nazi bit wasnt true but the
rest was. Its not a criminal offence hes committed but its sort of immoral/perverted conduct. He brings a
claim for breach of confidence. (Or misuse of private information rather)
Judgment:
Eady J says that exposing perverted conduct is not justifiable in this balance between expectations of
privacy and freedom of speech as it isnt in the public interest to have a persons private, legal (albeit
strange or perverted) sexual habits exposed to the public. (though it could be in the public interest if the
behaviour involved the mocking of jews or the holocaust)
Notes:
Further, if you look at the most recent cases particularly Murray v Sunday Express, the courts keep saying
you need to weigh all the factors up and that you cannot point to a decisive or conclusive factor. The
factors you weigh up would include things like what was previously said or done, the nature of the
invasion, etc

(4) The influence of the European Convention on Human Rights


Art 8: Right to respect for private and family life
A broader question on the role of HR jurisprudence in this area
In relation to the Human rights act, courts are clear that if a PA violated your rights, youd be protected.
However, there is this issue of horizontal effect which the courts havent quite decided yet. For instance,
Campbell couldnt rely on the HRA as MGN isnt a public body. However, theres an argument which
says that courts are public bodies and as such, have to give effect to individuals convention rights. The
argument means that a court hearing a case between 2 private individuals has to give effect to convention
rights.
This is very much an academic argument and the courts still havent been able to or wanted to decide
whether its true or not.

What do the courts say about the role of convention nights in relation to this tort of misuse of private
information?
They may be to some extent persuaded by Strasbourg but there are passages in some judgments which
suggest that its almost a happy coincidence that English law is being developed at the same time that art8
rights are being developed in Strasbourg jurisprudence. One future question we might think about is how
to engage the Strasbourg jurisprudence with English rules grown out of tort of breach of confidence.
Peck v UK (2003) 36 EHRR 41
Facts: The applicant was filmed in a public street on CCTV attempting to commit suicide by slashing his
wrists. Some six months later, the council published pictures of the incident in a report on the preventative
benefits of CCTV where the applicants face was not masked. Extracts of the footage were also shown on
regional TV though there, the applicants face was masked at the councils request. The applicant sought
JR of the councils decision to release the footage. This was refused along with an appeal. He then applied
to the ECHR.
Judgment:Following the disclosure of the CCTV footage, the applicant's actions were seen to an extent
which far exceeded any exposure to a passer-by or to a security observation and to a degree surpassing
that which the applicant could possibly have foreseen. The disclosure by the Council therefore constituted
a serious interference with his right to respect for private life. There were not relevant or sufficient
reasons to justify the disclosure by the Council without obtaining the applicant's consent or ensuring as
far as possible that his identity was masked. The applicant's subsequent media appearances did not
diminish the serious nature of the interference. Accordingly, the disclosures by the Council were not
accompanied by sufficient safeguards and they therefore constituted a disproportionate interference with
the applicant's rights under Article 8.
Notes:So here we note the ECHR's acceptance that certain incidents which take place in public can still
fall within someone's private life and for which they can have a reasonable expectation of privacy
Von Hannover v Germany (2004) 40 EHRR 1
Facts: Its a claim by the princess of Monaco against Germany on the basis that German law does not
adequately protect her privacy. She was photographed doing things such as eating in a private restaurant
(in English law, wed probably regard that activity as having a reasonable expectation of privacy), picking
up her children from school (that part however triggers the children elements and it may be that for kids
theres a higher expectation of privacy in both English law as well as Article 8 Strasbourg jurisprudence),
and also, while shopping and being at a public swimming pool. (In English law, you wouldnt regard
claimants as having a reasonable expectation of privacy when photographed in the streets going to shop
or a public place)
Judgment:Yet, in Von Hannover, the state was held to have violated her legal rights by failing to have
adequate protection against the publication of such photographs. (i.e. even the ones of her in public
violated art. 8)
Notes:The point is that even though English law has made significant efforts to align itself with the EU
case law, its still not brought itself into perfect symmetry with it.

So in the future, inevitably, some kind of decision is going to have to be made about what happens in
these kinds of cases. Particularly in Campbell itself, Hale says a picture of Campbell going to the shops
for a bottle of milk wouldnt count as misuse of private information which is exactly contrary to whats
said in Von Hannover.

(5) Statutory torts/remedies


Protection from Harassment Act 2007
Thomas v News Group [2002] EMLR 78
Data Protection Act 1998, ss 1-2, 13, 32

Seminar 15: Vicarious liability


(1) Requirements
D1 has committed a tort
D1 is D2s employee (and not an independent contractor)
D1s tort was committed in the course/scope of his employment

(2) Establishing an employment relationship: integration test


Notes:
To tell if someone is in biz on their own account, we have to look at 6 different factors.
1.
2.
3.
4.
5.
6.

Control
Provision of equipment
Provision of helpers
Degree of financial risk
Responsibility in investment and management
Opportunity to profit from sound management (i.e. being in a position where you make big
strategic decisions)

Market Investigations Ltd v Social Security Minister [1969] 2 QB 173


Facts:The particular employee worked as a market researcher. In order to do this, she conducted
interviews with a range of people. Market investigations co provided her with questions they wanted her
to ask. Told them what order to ask in and also gave information on how to extract more information from
an interviewee.

She did however have a degree of flexibility about when to do the work. Simply told to do a certain
number of interviews by a certain time.
Judgment:Held she was an employee specifically because of the very control the market investigations
company exercised over how she did the job. (Questions, order, methods)
Lee Ting Sang v Chung Chi-Keung [1990] 2 AC 374
Facts:Concerned a stone mason who worked for a particular subcontractor on short-term jobs. He
provided no equipment. He was given tools to work with when he got to the building site and he provided
no helpers either. Nor was he responsible for investment and management.
Judgment: held that he was an employee. The emphasis there was on the fact that he did not provide
equipment and had no responsibility for providing helpers or substitutes if he was ill.
Hall (Inspector of Taxes) v Lorimer [1994] 1 All ER 250
Facts:D was a vision mixer. Worked about 800 days over a 4 yr period. It was a highly technical, job with
regards to production of TV programmes and he worked on a range of projects.
He was obviously being told what jobs to do but as a highly skilled individual, he wasnt being told how
to do it. He didnt provide own equipment or helpers.
But, if the production companys customer failed to pay the production company for the project hed
worked on, he wouldnt get paid. He was therefore exposed to financial risk.
Judgment: That factor was held to be decisive in this particular context in making him an independent
contractor.
Viasystems (Tyneside) Ltd v Thermal Transfer (Northern) Ltd [2006] QB 510 (Concerning a
borrowed employee)
Facts:Viasystems contracted with Thermal Transfer to install air conditioning in its factory. Thermal
Transfer subcontracted ducting work to S & P, who then subcontracted with Cat Metalwork for the
provision of fitters and fitters mates, on a labour-only basis. A fitter and a fitters mate were supplied by
Cat Metalwork and worked under the supervision of a fitter from S & P. The fitters mate negligently
caused a flood in the factory. At first instance, only Cat Metalwork was vicariously liable. They appealed
contending Thermal Transfer should be vicariously liable as well.
Judgment: The CA held that it was possible for dual vicariously liability to exist. They said it is not
necessary for the borrower to have a direct (contractual) relationship of employment with the
borrowed/ employee.

May LJ: Concentrate on the relevant negligent act and see whose responsibility it was to prevent
it.
Who was entitled and in theory obliged to control the mate? Test is entitlement to control, with
an implied obligation to control.
The test in respect to a particular act is capacity to control the party to whom it is fair to
impose VL on, despite the liability flowing from the employee

Rix LJ: control is important, but not sufficient as a sole test. Consider also integration of the
employee into the work business or organisation of the employers. This is because VL involves
no fault on behalf of the employer.

(3) Course of employment: close connection test


Textbook by Salmond and the unauthorised mode test.
There were difficulties between distinguishing what counts as an unauthorised mode of doing the job and
doing something entirely different.
Rose v Plenty [1976] 1 WLR 141
Facts: The defendant milkman (and indeed all milkmen) had been told not to employ children to help
them on their rounds. D disregarded the instructions and had hired a 14-year old boy (C) to help him. Due
to negligent driving by D, C was injured.
At first instance, recovery from the employer was barred as it was decided that it was not even in the
scope of Ds employment to hire children and thus, there couldn't be vicarious liability.
Judgment: CA said that the scope of the job was to distribute the milk, collect the money and to bring
back the bottles to the van (Denning) and thus, the employment of C was merely an unauthorised mode
of going about the job rather than outside the scope of Ds employment.
Going beyond the unauthorised mode test?
More recent cases leading up to Lister started to go beyond the unauthorised mode test. The key case is a
decision of the Canadian SC.
Bazley v Curry [1999] 2 SCR 534
Facts: C had been taken into care by a charitable institution. He was then sexually abused by one of the
employees of the institution.
Judgment:Canada SC held that the employer was vicariously liable for that deliberate sexual abuse of the
boy. They admitted you couldn't justify this by saying what the employee was doing was trying to further
the business of the employer. Indeed, he was doing the opposite.
But they said there are 2 justifications for imposing VL,
-

Fair compensation (benefit-burden); recognised the charity issue but said you can also
characterise this in terms of the employers organisation having created the risk of the tort being
committed.
Deterrence
o As far as deterrence was concerned, SC said its important to impose VL so as to give
very incentive and encouragement to employers so as to prevent sexual abuse happening
in regard of their business
o Greater care in training, recruitment, supervision etc

This kind of theoretical explanation of VL was found convincing by HL in Lister


Lister and the Close Connection Test
Lister v Hesley Hall Ltd [2002] 1 AC 215
Facts: The claimants were residential students at a school for difficult children owned by the defendant.
One of the wardens employed by the defendant was sexually abusing the children in his care and was
eventually subjected to criminal proceedings. The claimants sought to hold the defendant vicariously
liable for the harms they suffered as a result of the abuse.
Judgment: The HL adopted the closeness of connection test to determine whether an intentionally
wrongful act by an employee would fall within the course of employment. Here the sexual abuse occurred
on the employers premises whilst the employee was engaged in performing his duties of caring for the
children. As such, there was a close connection between the employment and the abuse so the employer
would be vicariously liable. This was particularly so as there was an obvious risk of sexual abuse in the
circumstances so the employer should have been alert to it.
Notes:
Steyn the need for the employment to be inextricably linked with the tort
Clyde Take a broad approach considering all factors (nature of the act, purpose, context)
-

The fact that the act is done on the employers premises during working hours is helpful but not
conclusive
Acts of passion and resentment will often fall outside the scope of employment
(+Millet) The mere fact that the employment has created the opportunity for the tort isnt enough

Hobhouse (dissent on close connection test) idea of a non-delegable duty


Following this expansion of liability, employers have been found liable in subsequent cases for intentional
torts of their employees. In Mattis v Pollock. vicarious liability was found where a bouncer, intent on
revenge, stabbed a patron of the night club he worked at. Dubai Aluminium Co Ltd v Salaam.[18]
established liability for fraud of employees, where it is outside of their duties or authority to make certain
representations.
Dubai Aluminium Co Ltd v Salaam [2003] 2 AC 366 [1]-[36] (Lord Nicholls)
Mattis v Pollock [2003] 1 WLR 2158
So even though it was an act of resentment/revenge, considering all factors (shows the importance of this
point from Lord Clyde), because hed been encouraged by the employer to act in this kind of way, VL
was recognised.
Majrowski v Guys and St Thomas NHS Trust [2007] 1 AC 224
-

Clarifies that VL extends to equitable wrongs and breaches of statutory obligations (unless
specifically excluded)

It Is perfectly possible to have VL for harrassment

Seminar 16: Remedies


(1) Compensatory damages
(A) THE PURPOSE OF COMPENSATORY DAMAGES
Livingstone v Raywards Coal Co (1880) 5 App Cas 25, 39 (Lord Blackburn): the measure of damages
is that sum of money which will put the party who has been injured, or has suffered, in the same position
as he would have been in if he had not sustained the wrong for which he is now getting his compensation

Smith New Court Securities Ltd v Scrimgeour Vickers (Asset Management) Ltd [1997] AC 254: in a
case of deceit or negligent misrepresentation C put into as good a position as if no representation had
been made, but not into as good a position as if the statement had been true
(B) POSSIBLE TORT ACTIONS
(i) Action by living claimant
(ii) Action by administrator of deceased claimants estate suing in deceased claimants name and proving
elements of the claim that the deceased would have had to prove if he were still alive and suing himself:
Law Reform (Miscellaneous Provisions) Act 1934, s 1(2)
(iii) Action by dependants of deceased suing in their own names to recover the economic loss which they
personally suffer because deceased will no longer provide for them, but NB their action is also derivative
in the sense that they must prove the elements of the claim that the deceased would have had to prove if
he had survived and was now suing for personal injury: Fatal Accidents Act 1976
(C) DAMAGES FOR PERSONAL INJURY
(i) Lump Sums or Periodical Payments?
Courts Act 2003, s 100, inserting new Damages Act 1996, s 2
Damages Act 1996 s2Periodical payments (as amended by Courts Act 2003 s100)
(1) A court awarding damages for future pecuniary loss in respect of personal injury
(a) may order that the damages are wholly or partly to take the form of periodical payments,
and.
(b) shall consider whether to make that order.
(2) A court awarding other damages in respect of personal injury may, if the parties consent, order
that the damages are wholly or partly to take the form of periodical payments.

(3) A court may not make an order for periodical payments unless satisfied that the continuity of
payment under the order is reasonably secure.
(4) For the purpose of subsection (3) the continuity of payment under an order is reasonably secure if

(a) it is protected by a guarantee given under section 6 of or the Schedule to this Act,.
(b) it is protected by a scheme under section 213 of the Financial Services and Markets Act
2000 (compensation) (whether or not as modified by section 4 of this Act), or.
(c) the source of payment is a government or health service body.
(to sum up s2(4), continuity of payment is reasonably secure where payment is made by government or a
local authority. Private defendants security of payment will be established if the private defendant
purchases an annuity)
E.g. Tameside and Glossop Acute Services NHS Trust v Thompstone [2008] 1 WLR 2207 (The 4
claimants were left brain-damaged after suffering birth-asphyxia as a result of the admitted defendants
negligence. A periodical payment order was made)
(ii) Financial/Pecuniary loss
Need to distinguish between two types of loss. Past financial loss (No significant legal difficulties. You
can typically recover in full) and future financial loss. (Calculated by multiplying the multiplier by the
multiplicand)
The multiplier/multiplicand method

Multiplier: figure representing the number of years the loss will continue

Multiplicand: figure representing the annual loss (net of deductions) the claimant will suffer

Complicating factors

vicissitudes of life
acceleration element
inflation: Wells v Wells [1999] 1 AC 345
the lost years: Pickett v British Rail Engineering [1980] AC 136

Vicissitudes of life
Bad things might happen to C in the future (reduction in life expectancy or general ability to ensure life at
the standard assumed); these are termed the vicissitudes of life and require that the figure be reduced.
Courts and personal injury lawyers use actuarial tables from people concerned with these life expectancy
statistics to determine this.
Acceleration Element
The idea that part of the award can be investment and this should count towards the total award.
Inflation: Wells v Wells [1999] 1 AC 345

Wells v Wells [1999] 1 AC 345


The question for the court was how to adjust the award for acceleration and inflation.
Accelerated receipt: HL said you assume C will invest their damages in index linked government
securities. Theyre adjusted for inflation and they also give a guaranteed return. (3% interest at the time of
the case)
Thats a very cautious way to invest money. But HL said its appropriate to assume such a cautious
approach being taken by C. because, a C cant afford to gamble with the money he needs for basic
necessities and medical care.
So, in response to the Wells decision, parliament in the Damages Act 1996 (as amended after wells) fixed
the assumed rate of return at 2.5%. (At the time they passed the statute in 2007, thats what index linked
government securities were paying)
So today, courts assume people will invest in index linked government securities paying 2.5%. In fact
today though these securities pay <1%. So the reality doesnt reflect what courts are assuming.
The lost years: Pickett v British Rail Engineering [1980] AC 136
Taking compensatory awards literally, itd mean C shouldnt be compensated for a loss of life expectancy;
Pickett v British Rail Engineering [1980] AC 136
HL decided they didnt like a literal mathematical application of multiplier rule. So they said that for the
period you would have been alive, but now wont be, you can recover the amount of your earnings that
would have been spent on your family and your dependants.
In order to calculate future financial loss accurately, many complicated mathematical calculations have to
be done. Were not so concerned with it here. In practice, personal injury lawyers rely on a book called
the Ogden Tables which set out what the appropriate figures are year by year for Cs of particular ages.
(iii) Non-financial loss

pain and suffering subjective assessment


loss of amenity (not being able to do things you enjoyed) objective assessment

H West & Son v Shephard [1964] AC 326 &Lim Po Choo v Camden & Islington AHA [1980] AC 174
Both cases concerned with people who as a result of Ds negligence went into a coma. By its very nature,
its clear there isnt much pain and suffering (besides the initial impact).
However, the Loss of Amenity award being assessed objectively, on the basis of what youve actually
been deprived of, would be high. Because of course in a coma, you cant do anything.
So you can get non-pecuniary damages for pain and suffering and loss of amenity, but how do you value
it?

There are 2 main books.


-

Kemp and Kemp on personal injury damages. Created to try and get judicial consistency.
Judicial bodies something guidelines on something (check) (for np damages)

So we have a way of ascertaining what judges have awarded in previous cases. But that doesnt
necessarily reflect what people place on loss and suffering.
Heil v Rankin [2001] QB 272
LC in previous report had proposed that damages for non-pecuniary injury should be increased
significantly. Part of its argument for doing so was that in Northern Ireland where jury awards were still
used till the 80s, non-pecuniary damages were a lot higher. The LC said that this illustrates that the
English courts damages awards have sort of fallen out of sync/failed to keep up with public perceptions
about the value of pain and suffering.
CA in Heil said it might be right so we shall increase awards but not as much as LC wanted. For small
awards (under 10k), no increase. For awards at the top, well increase by a third. And for awards between
10k and the top (didnt get a figure but probably 6 figures) but the increase should be tapered.
At [99], they say theres no need to do this again. This is a one-off. That could be right, maybe theres no
need to do it again. But if you look at the case, its quite interesting to see what they say about Northern
Ireland. Namely, that jury awards, and ordinary peoples perception of pain and suffering is a lot higher
than the judges perception of what it should be.
(D) AGGRAVATED DAMAGES
To compensate for additional pain, suffering, and distress, caused by the manner in which the tort was
committed. Initially had a narrow scope. E.g. Richardson v Howie [2005] PIQR Q3 but has since been
widened.
Rowlandson v Merseyside Police [2007] 1 WLR 1065
Facts:C was being disturbed by noisy neighbours and called the police. When they arrived, for some
reason there was an altercation between the policeman and C. The officer clapped C in handcuffs in front
of her family, took her into police car and to the station. She was then charged with obstructing a police
officer. A charge she was then acquitted of and it was found that the officer invented the supposed events.
Judgment: CA awarded damages for the torts against her, including the battery in dragging her to the car.
They held that aggravated damages could be awarded. Her anxiety caused by the tort had been increased
by the way the tort had been committed.
Notes:This case indicates the potential scope of an award of AD.
Finally, it is worth reiterating out that aggravated damages are a form of compensatory damages.
Choudhary v Martins [2008] 1 WLR 617

C was repeatedly harassed etc amongst other things by D. At trial, the judge made an award for
psychiatric injury and a separate award for injury to feelings which reflected the aggravating aspects of
the case.
This suggests that normally, you can just bundle up the aggravated damages with other non-pecuniary
damages. So often, you can just make one award which includes all the non-pecuniary loss under one
heading.

(2) Non-compensatory damages


(A+ B) NOMINAL DAMAGES AND CONTEMPTUOUS DAMAGES
Nominal An award for no loss with regards to a tort actionable per se (battery, defamation)
Contemptuous where C has technically suffered loss due to a tort but the court (or jury in defamation) is
trying to express its contempt for the claim being brought.
Grobbelaar v News Group Newspapers Ltd [2002] 1 WLR 3024

The claimant was the subject of accusations of match fixings, and brought an action for
defamation.
Held that the claimant was entitled to 1 damages, the lowest sum possible.

(C) EXEMPLARY DAMAGES (PUNITIVE )


Rookes v Barnard [1964] AC 1129
Exemplary damages are anomalous in tort but have too much history to get rid of it.
However, we can confine it to 2 categories.
i.
ii.

Where D is a public body/authority and acted in an arbitrary and oppressive manner in


committing the tort.
Where D calculates that if he commits the tort and has to pay compensatory damages, he will still
make a profit.
o Particularly suited to defamation; highlighted by Cassell v Broome [1972] AC 1027
o Concerned publication of book. HL confirmed that exemplary damages where D
calculated hed make a profit for committing tort were appropriate. Also said that a court
had to be satisfied before making an exemplary award that a compensatory award wasnt
accurate.

Kuddus v Chief Constable of Leicestershire [2002] 2 AC 122


Clarifies that exemplary damages can be recovered even for torts where it hadnt been recognised before
1964 as long as It fits into one of the 2 categories.
(D) RESTITUTIONARY DAMAGES

a monetary remedy that is measured according to the defendant's gain rather than the plaintiff's loss. The
plaintiff thereby gains damages which are not measured by reference to any loss sustained. In some areas
of the law this heading of damages is uncontroversial; most particularly intellectual property rights and
breach of fiduciary relationship.
The basis for restitutionary damages is much debated, but is usually seen as based on denying a
wrongdoer any profit from his wrongdoing. The really difficult question, and one which is currently
unanswered, relates to what wrongs should allow this remedy.
Ministry of Defence v Ashman (1993) 66 P & CR 195
Abstract: A person entitled to possession of land may claim against a person who has occupied it without
consent damages assessed on a restitutionary basis, by reference to the proper letting value of the property
for the relevant period. D2, a Flight Sergeant in the RAF, was permitted by P to occupy married quarters
with his wife D1 and their two children and signed a certificate that he was entitled to occupy the property
only so long as he remained a serving member of the RAF living with his spouse, and that he would be
required to remove his family from the accommodation if he ceased to live with his spouse. In February
1991 D2 moved out, leaving D1 and their children in possession. They were given notice on March 14,
1991 to vacate by May 16, 1991, but did not do so because they had nowhere else to go. At the time D2
moved out his quartering charge was GBP 95.41 a month; but on May 17, 1991 a seven-day notice was
served on D1 asserting for the first time the right to claim damages for trespass at GBP 108.93 per week.
In November 1991 the judge made an order for possession and adjourned the claim for mesne profits. In
March 1992 one of Ds' children got married and left home, and D1 received an offer from the local
authority of a two-bedroom flat at GBP 33.44 a week which she accepted. She vacated the married
quarters on April 26, 1992. At the hearing of the claim for damages evidence was given for P that a
quartering charge is fixed by starting with a notional rent calculated by reference to a national average of
local authority housing rents and then deducting 42 per cent; that where there is unauthorised occupation
P seeks to recover on the basis of a market rent; and that the market rent in this case was GBP 108.93 per
week. The judge noted that he found the evidence of open market rent "totally compelling and
persuasive", but found that, in taking possession of the married quarters, P had acted on a clear and
unequivocal assurance that the charge was calculated by reference to the average of local authority rents,
and that it was not open to P to disregard that assurance and seek to claim a higher rent.
Held, allowing P's appeal and remitting the assessment of damages to the county court, that (1) the
finding of estoppel could not be sustained; (2) L was entitled to recover damages assessed on a
restitutionary basis by reference to the proper letting value of the property to D1 for the relevant period;
and (3) whilst in an ordinary case and a free market the proper letting value of the property would be the
value to the trespasser of its use, such value could in the unusual circumstances of the present case
reasonably be assessed by reference to what D1 would have had to pay for suitable local authority
accommodation

(3) Injunctions
(A) TYPES OF INJUNCTION
(1) final/perpetual (given at the conclusion of a trial. Affects the future)

(2) interim (essentially to keep everything the same until trial)


(3) mandatory and prohibitory
(4) quia timet (= because s/he fears) (because you fear D will commit the tort against you. So its
given to stop the act that will amount to the tort)
(B) G ENERAL PRINCIPLES
(i) inadequacy of common law remedy
E.g. damages may not be adequate for on-going events (e.g. nuisance)
(ii) clean hands: Measure Bros Ltd v Measures [1910] 2 Ch 248 (general principle of equity)
(iii) delay & acquiescence
(iv) equity will not act in vain: AG v Observer Ltd [1990] 1 AC 109(the book spycatcher was easily
available in the US and even reviewed in other English newspapers)
(v) public interest
Dennis v Ministry of Defence [2003] 2 EGLR 121 (the case concerning the person who lives near an
airfield) highlights the way the courts are taking account of the public interest in an activity continuing to
not grant an injunction
Miller v Jackson [1977] QB 966 The public interest, which requires young people to have the benefit of
outdoor games, may be held to outweigh the private interest of neighbouring householders who are the
victims of the balls therefrom, so as to prevent the grant of an injunction, even if the sportsmen are guilty
of negligence.
Kennaway v Thompson [1981] QB 88
The case concerned powerboat racing in the lake district. Here, the court allowed the continuation of
some powerboat racing but limited the number of races that could be held each year. It even limited the
number of races that could be held involving the boats with the biggest engines which were particularly
noisy.
This case is particularly useful as it highlights how the courts have the power to tailor injunctions to grant
whats called an injunction on terms where you allow the nuisance creating activity to continue but only
to a certain extent or at particular times on in a particular way; so its possible for courts to be quite
creative in deciding how to use injunctions.
(C) SPECIAL RULES FOR DEFAMATION
Bonnard v Perryman [1891] 2 Ch 269e
A case where a 7 member CA, specially convened for the purpose holds that the importance of Freedom
of Speech is such that interim injunctions for defamation (injunctions preventing publication until trial)
should not be permitted in cases where D pleads a valid defence. (e.g. truth, justification, qualified
privilege) or, where D contests that the words have a defamatory meaning.

So that in effect makes it very difficult to get an injunction to restrain publication of defamatory material.
So if D then loses at trial, court can of course award a final injunction stopping publication ever again.

Вам также может понравиться